Download as pdf or txt
Download as pdf or txt
You are on page 1of 299

A. CARBOHYDRATES 1.

Describe and classify carbohydrates Description Contain C, H and O molecules Contain a C=O (ketone) and an OH(aldehyde) functional group Classification Based on certain properties The size of the base carbon chain Location of the CO functional group Number of sugar units Stereochemistry of compound Chemical Properties Some ( not all ) carbs are reducing substances (donate electrons) Chemical reduction of other substances These sugars must contain an aldehyde or ketone group Reducing sugars o Glucose o Maltose o Lactose o Fructose o Galactose Sucrose is not a reducing substance

2.

Describe carbohydrate metabolism Glucose is primary energy source Nervous tissue cannot concentrate or store carbohydrates, so a steady supply of glucose is needed Once the level of glucose falls below a certain range, normal function is impaired Carbohydrate Breakdown Ultimate Goal o Convert glucose to CO2 and water with ATP as a by-product Discuss glycolysis as it pertains to carbohydrate metabolism and carbohydrate detection methods Hydrolysis of glucose by an enzyme into pyruvate or lactate; This process is anaerobic

3.

4.

Fasting blood glucose levels Hyperglycemic o Fasting blood glucose > 100 mg/dL Hypoglycemic o Fasting blood glucose < 50 mg/dL

Page 1

5.

Describe glycolysis Glycolysis the conversion of glucose and other hexoses into lactate or pyruvate Breakdown of glucose for energy production Describe carbohydrate breakdown

6.

Ultimate Goal o Convert glucose to CO2 and water with ATP as a by-product o Possible channels o Converted to liver glycogen and stored o Metabolized to CO2 and H2O o Converted to keto-acids, amino acids, and proteins o Converted to fats and stored in adipose tissue Biochemical pathways in carbohydrate breakdown o Embden-Meyerhoff pathway o Converts glucose to pyruvate/lactate o Primary energy source for humans o Hexose monophosphate shunt o Oxidizes glucose to ribose and CO2 o Produces NADPH as an energy source o Glycogenesis o Converts glucose to glycogen

7.

Describe the role of the liver in maintenance of glucose levels. The liver maintains the glucose levels by: o Glycogenesis Converts glucose to glycogen Page 2

Glycogenolysis Breakdown of glycogen to form glucose Glycogenolysis occurs when plasma glucose is decreased Occurs quickly if additional glucose is needed Controlled by hormones & enzymes Gluconeogenesis Formation of glucose from non-carbohydrate sources, such as amino acids, glycerol & fatty acids into glucose Occurs mainly in the liver During long fasts, gluconeogenesis is required to maintain blood glucose levels because glycogen stores are up in about 24 hours

During a fast, the blood glucose level is kept constant by mobilizing the glycogen stores in the liver Page 3

8.

What hormones does the liver use to maintain glucose levels? Insulin o Produced by the beta cells of the islets of Langerhans in the pancreas o Promotes the entry of glucose into liver, muscle, and adipose tissue to be stored as glycogen and fat; o Inhibits the release of glucose from the liver o Insulin secretion controlled by: Blood glucose level Certain Amino Acids ie. leucine, & arginine

Glucagon o Secreted by the alpha cells of the pancreatic islets of Langerhans o Increases blood glucose by stimulating glycogenolysis and gluconeogenesis o 2nd most important glucose regulatory hormone o Referred to as a hyperglycemic agent o Synthesized in alpha cells of the islets of Langerhans o Action/Effect of o Stimuli decreased plasma glucose o Action Increases glycogenolysis & gluconeogenesis Promotes breakdown of fatty acids Promotes breakdown of proteins to form amino acids Increases plasma glucose concentration Somatostatin o Origin-Delta cells of the islets of Langerhans in the pancreas o Effect - increase plasma glucose o Actions antagonistic to insulin, Page 4

inhibits endocrine hormones including glucagon & growth hormone Inhibits secretion of insulin, glucagon, and growth hormone resulting in an increase in plasma glucose levels

Other regulatory hormones o Epinephrine One of two glucose regulating hormones from the adrenal gland Origin adrenal medulla Action/effect Inhibits insulin secretion & release Promotes lipolysis Stimulates glycogenolysis Immediate release of glucose Stimuli Neurogenic - based on physical / emotional stress. Adrenal tumors o Glucocorticoids - such as cortisol Origin adrenal cortex Effect antagonistic to insulin increases blood glucose promotes gluconeogenesis from breakdown of proteins inhibits the entry of glucose into muscle cells Stimuli anterior pituitarys ACTH o Growth Hormone (GH) and Adrenocorticotropic Hormone (ACTH) Origin anterior pituitary gland Effect antagonistic to insulin Increases plasma glucose levels inhibits insulin secretion inhibits entry of glucose into muscle cells inhibits glycolysis inhibits formation of triglycerides from glucose Stimuli decreased glucose stimulates its release increased glucose inhibits its release o Thyroid hormones (such as thyroxine) Origin thyroid gland Effect Increases absorption of glucose from intestines Promotes conversion of liver glycogen to glucose o Stimuli pituitary glands TSH

9.

What hormones does the pancreas produce that regulate carbohydrate metabolism? Insulin o Produced by the beta cells of the islets of Langerhans in the pancreas o Promotes the entry of glucose into liver, muscle, and adipose tissue to be stored as glycogen and fat; Page 5

o Inhibits the release of glucose from the liver Glycagon o Secreted by the alpha cells of the pancreatic islets of Langerhans o Increases blood glucose by stimulating glycogenolysis and gluconeogenesis Somatostatin o Synthesized by the delta cells of the pancreatic islets of Langerhans o Inhibits secretion of insulin, glucagon, and growth hormone resulting in an increase in plasma glucose levels

10.

What impact does cortisol, catecholamine hormones and thyroid hormones have on glucose levels. Cortisol o o o

Secreted by the adrenal glands; Stimulates glycogenolysis, lipolysis, and gluconeogenesis Increases plasma glucose by decreasing intestinal entry into cells and increasing gluconeogenesis. Epinephrine o Increases plasma glucose by inhibiting insulin secretion. o Secreted by the medulla of the adrenal glands. o It stimulates glycogenolysis and lipolysis; o It inhibits secretion of insulin. o Physical or emotional stress causes increased secretion of epinephrine and an immediate increase in blood glucose levels. Thyroid hormone o Secreted by the thyroid gland; o Stimulates glycogenolysis and gluconeogenesis; o Increases glucose absorption from the intestines

11.

Describe the metabolic defect in Diabetes Mellitus? Glucose does not get into the cells

12.

What are typical glucose levels, insulin levels, and ketone levels in this disease? 1) Type I Diabetes Mellituso Glucose levels are increased o Insulin levels are decreased o Ketones present 2) Type II Diabetes Mellitus o Glucose levels are increased o Insulin levels are normal to decreased o Glucagon response is decreased o No ketones present.

13.

What is glycosylated hemoglobin? Glycated hemoglobin is formed from the nonenzymatic, irreversible attachment of glucose to hemoglobin A1. Page 6

14.

Measurement of glycated hemoglobin reflects blood glucose levels for the past 2 3 months. It is useful in monitoring effectiveness of treatment and compliance of diabetic individual to treatment protocol. The primary determinant in the rate of hemoglobin A-1c synthesis is the life span of the Red Blood Cell and the level of average glucose concentration

What are the normal glucose levels in a fasting individual? Normal glucose level (fasting)- 70 to 110 mg/dl

15.

What are panic or critical glucose values? Panic Values- >126 (fasting), > 200 (random or glucose tolerance test)

16.

Describe the relationship between glucose levels in urine and serum. Glucose will never be found in urine unless the serum glucose is high enough to exceed the renal threshold and spill over into the urine Glucose is filtered by the glomeruli, reabsorbed by the tubules, and normally not present in urine. If the blood glucose level is elevated, glucose appears in the urine, a condition known as glucosuria. An individuals renal threshold for glucose varies between 160 and 180 mg/ dL. When blood glucose reaches this level or exceeds it, the renal tubular transport mechanism becomes saturated, which causes glucose to be excreted into the urine.

17.

Name inherited disorder of carbohydrate metabolism Glucose 6 phosphatase deficiency AKA von Glerke Galactosemia o Characterized by a deficiency or absence of galactokinase, o Enzyme defect prevents metabolism of galactose. o Galactose is found in milk as a component of lactose, with galactosemia generally identified in infants. Why should serum for glucose be removed from the red cells as soon after collection as possible? Serum should be removed from RBCs ASAP because the cells will use the glucose and falsely decrease the glucose level.

18.

19.

What anticoagulant preservatives are used for glucose specimens? Why? Sodium fluoride is the anticoagulant of choice because it inhibits glycolytic enzymes.

Page 7

20.

What is a two-hour post prandial glucose? Why is it performed? This is a sample taken two hours after eating and can determine how well the body is using the glucose. This sample will show insulin function which is the main reason it is performed.

21.

Why are D-xylose tolerance tests performed? This test is done to differentiate malabsorption from pancreatic insufficiency.

22.

What is the normal range of CSF glucose? Normal CSF glucose- 40-70 mg/dl CSF glucose is 2/3 of the plasma glucose which is due to the glucose entering the CSF by facilitative transport. The carrier mechanism is responsible for transporting glucose across the downhill gradient. Meningitis can cause a change in the CSF glucose levels.

23.

Describe states that result in alteration of serum glucose. Diabetes mellitus PancreatitisRecent meal intake Fasting Increases glucose (insulin is not working) Increases glucose (insulin not produced) Increases glucose Decreases glucose

24.

Lactate The normal end product of glucose metabolism is pyruvate; Lactate is produced under conditions of oxygen deficit (anaerobic metabolism). The production and accumulation of lactate in the blood and its measurement aid in assessing the degree of oxygen deprivation that is occurring. Change in the blood lactate level precedes a change in blood pH. Lactate is metabolized by the liver via gluconeogenesis. What are normal glycosylated hemoglobin ranges? Normal glycosylated hemoglobin range o 4.5-8.0 High values indicate that the patient has not been following the proper diet

25.

Page 8

Page 9

B. LIPIDS 1. Describe the cholesterol Function o Used to manufacture and repair cell membranes, o Used in synthesis of bile acids and vitamin D Synthesized in the liver and obtained from the diet. Precursor for synthesis of bile acids, steroid hormones, and vitamin D Transport mechanism o Transported through the blood by LDL (low density lipoproteins) Storage sites o Stored in the skin, adipose tissue, and muscle cells. Esterified cholesterol-2/3 of the total cholesterol is esterified Describe triglycerides Formed from one glycerol molecule with three fatty acid molecules attached via ester bonds Comprises 95% of all fats stored in adipose tissue Transport mechanism Transported through the body by chylomicrons and VLDL HDL-cholesterol Synthesized by the liver and by the intestine In normal lipid metabolism, HDL removes excess cholesterol from peripheral tissues and transports it to other catabolic sites providing an antiatherogenic effect.. What is the role of lipase? Lipase and bile acids are used to break down fats in lipid absorption. Found in pancreas, with lesser amounts in gastric mucosa, intestinal mucosa, adipose tissue Clinical significance: o Increased serum levels in acute pancreatitis occur in 4 8 hours after the onset of pain, with peak values in 24 hours, and return to normal in 8 14 days. List the bile acids Cholic acid, Glycocholic acid Taurocholic acid Where are bile acids synthesized? Liver What does the presence of bile acids in serum indicate? . The presence of bile acids in serum can indicate liver disease.

2.

3.

4.

5.

6.

7.

Page 10

8.

What are chylomicrons? Triglycerides are transported through the body by chylomicrons Chylomicrons- the largest lipoprotein particles with diameters ranging from 80-1200 nm. They are 90-95% triglycerides, 2-6% phospholipids, 2-4% cholesteryl ester, 1% free cholesterol, and 1-2% apolipoprotein. How does serum appear with increased chylomicrons? increased cholesterol? increased triglycerides? Increased Chylomicrons- a creamy layer appears on top of the serum. Increased Cholesterol- serum appears milky white Increased Triglycerides- serum appears turbid. Given a value of 38 mg/dl for the HDL, 140 for triglycerides and 210 for total cholesterol, calculate the LDL and VLDL. VLDL= triglycerides/5 LDL= total cholesterol- VLDL- HDL VLDL= 140/5 =28 LDL= 210-28-38= 144

9.

10.

C. LIVER FUNCTION AND HEME-DERIVATIVES 1. Describe liver function in reference to each of the following: 3) Carbohydrate synthesis and metabolism The liver uses glycogenesis to make glycogen from glucose, simple sugars and amino acids. The glycogen produced is stored and used as needed. The liver also uses glucose for maintenance of mitochondrial NADH and to generate ATP in the Embden Myerhoff pathway. 4) Protein synthesis and metabolism The liver makes various proteins (transferrin, prothrombin, and ceruloplasmin) which are used to transport materials like iron, and copper. The liver also uses the proteins for nutrition, to regulate oncotic blood pressure (albumin), and for coagulation. o These proteins include albumin, HDL, LDL, VLDL, haptoglobin, angiotensin, erythropoiten, and many others. o These proteins also are used for nitrogen excretion by processing ammonia, urea, creatinine and uric acid. 5) Lipid synthesis and metabolism This is where cholesterol, triglycerides and phospholipids are synthesized. Free fatty acids are metabolized in the citric acid cycle into NADH, and bile acids are produced form cholesterol. 6) Porphyrin synthesis The liver produces the enzyme aminolevulinic acid synthase which controls the synthesis of porphyrins which eventually form heme molecules. Page 11

7) Bile acid synthesis the bile acids are conjugated with amino acids to form bile salt. It is synthesized by cholesterol in the bile ducts and ends up in the intestine where the lipids are digested. 8) iron and vitamin storage Iron is stored in the liver and transported wherever needed by transferrin, and vitamins are stored in the liver and available to be used whenever needed. 9) Excretion of metabolic end product and detoxification It converts ammonia to urea. 10) Bile pigment formation Bilirubin is the principal pigment in bile and is derived from the breakdown of hemoglobin when aged red blood cells are phagocytized by the reticuloendothelial system, primarily in the spleen, liver and bone marrow. 2. Where is bilirubin produced Bilirubin is produced in the reticuloendothelial system from the breakdown of hemoglobin from senescent red blood cells Bilirubin forms a complex with albumin for transport to the liver. In this form, bilirubin is unconjugated and not water soluble. Describe heme catabolism by the reticuloendothelial system (extravascular hemolysis). List states associated with increased extravascular hemolysis. Hemoglobin is broken down extravascularly into globin and heme (iron and protoporphrin IX) Protoporphrin breaks down further into unconjugated bilirubin which is carried to the liver by albumin and conjugated with glucuronyl transferase. o This is associated with RBC membrane defects and defects in the heme structure. Identify the function of haptoglobin, hemopexin albumin and methemalbumin in heme catabolism in intravascular hemolysis. Haptoglobin binds free hemoglobin and takes it back to the liver so it is not lost in the urine, Hemopexin albumin removes circulating heme from the blood and delivers it to the liver, Methemalbumin is free heme oxidized and bound to albumin which is carried to the liver and acts as storage that can be used until enough hemopexin is produced and made available to the liver. List states associated with increased intravascular hemolysis. Intravascular hemolysis is associated with increased immunologic processes, mechanical injury, and toxins. Differentiate between unconjugated and conjugated bilirubin Conjugated bilirubin

3.

4.

5.

6.

Page 12

Bilirubin is conjugated in the hepatocyte endoplasmic reticulum with glucuronic acid to form bilirubin diglucuronide ( conjugated bilirubin). o The reaction is catalyzed by UDP o Conjugated bilirubin is water soluble. o Conjugated bilirubin is excreted into the bile for storage in the gallbladder, secreted into the duodenum in response to gallbladder stimulation, and reduced by anaerobic bacteria in the intestine to urobilinogen. o Some intestinal urobilinogen is reabsorbed; A portion returns to the liver and some enters the circulation for excretion in the urine, whereas the remaining portion in the intestines is oxidized by anaerobic bacteria for excretion in the stool as urobilin. o Urobilin is an orange- brown pigment that gives stool its characteristic color. Unconjugated bilirubin o Bilirubin is produced in the reticuloendothelial system from the breakdown of hemoglobin from senescent red blood cells o Bilirubin forms a complex with albumin for transport to the liver. o In this form, bilirubin is unconjugated and not water soluble. o

7.

Describe the process of bilirubin conjugation. What is the role of glucuronyl transferase and where is this enzyme synthesized? What relationship exists between enzyme synthesis and neonatal physiologic jaundice? Hemoglobin is broken down into portoporphrin which is converted into unconjugated bilirubin which is bound to albumin and carried to the liver where it is converted to conjugated bilirubin by the enzyme glucuronyl transferase (synthesized in the liver). What is Kernicterus? Why does it develop? This is a serious newborn condition that occurs in the central nervous system because of high bilirubin levels. It is caused by an under developed blood brain barrier, and because newborns do not produce enough glucuronyl transferase. Describe excretion of bilirubin and resulting formation of urobilinogen? Why is urobilinogen normally present in urine and serum? Conjugated bilirubin is taken to the intestines where bacteria convert it into urobilinogen. Urobilinogen is normally present in the urine because a small amount of it is filtered back to the liver where it is recirculated and sent to the kidney where it is excreted in the urine. Prehepatic jaundice Prehepatic jaundice occurs when there is excessive erythrocyte destruction, as seen in hemolytic anemias, spherocytosis, toxic conditions, hemolytic disease of the newborn caused by Rh or ABO incompatibility, etc Page 13

8.

9.

10.

In these cases, the rate of hemolysis exceeds the livers ability to take up the bilirubin for conjugation. Prehepatic jaundice is characterized by an increased level of unconjugated bilirubin in the serum.

11.

Hepatic jaundice occurs when the liver cells malfunction and cannot take up, conjugate, or secrete bilirubin. o Gilbert syndrome: Defect in the ability of hepatocytes to take up bilirubin; due to transport problem of bilirubin from the sinusoidal membrane to the microsomal region; characterized by mild increase in serum level of unconjugated bilirubin o Neonatal physiological jaundice: Level of UDP- glycuronyltransferase is low at birth; Takes several days for the liver to synthesize an adequate amount of the enzyme to catalyze bilirubin conjugation; causes increased serum level of unconjugated bilirubin Posthepatic jaundice Occurs when an obstruction blocks the flow of bile into the intestines. This is referred to as extrahepatic cholestasis and may be caused by gallstones obstructing the common bile duct, neoplasms such as carcinoma of the ampulla of Vater or carcinoma of the pancreas, and inflammatory conditions such as acute cholangitis or acute pancreatitis. Posthepatic jaundice is characterized by: o Significantly increased level of conjugated bilirubin in serum, o Increased level of unconjugated bilirubin in serum, o Increased conjugated bilirubin in the urine, o Decreased urine and fecal urobilinogen o Stool that appears pale in color. Why are bilirubin determinations performed on amniotic fluid?

12.

13.

D. 1.

B. Porphyrins and Heme Derivates What are the porphyrias? These are deficiencies in the enzyme production that can be acquired or inherited. They result in increased production of one of the heme precursors to which they are intermediates. What is the physiologic function of porphyrins. The most important function of porphyrias are to chelate iron from heme.

2.

Page 14

3.

When are plasma hemoglobin levels increased and how are they measured? The plasma hemoglobin levels are increased during thalessemias and hemoglobinopathies. They can be measured by cellulose acetate electrophoresis or citrate agar electrophoresis. What reagent is used for the detection of urobilinogen? When would urobilinogen levels be decreased? Increased? Ehrlrich's reagent is most often used for urobilinogen detection Increased Urobilinogen levels o Excess hemolysis, o Liver damage by hypoxia o Exposure to various toxic agents. Decreased Urobilinogen levels o Obstructive jaundice because there is a limited delivery of bilirubin to the gut. When would myoglobin be increased? How are serum myoglobin levels measured? Myoglobin is increased when there is trauma to skeletal or cardiac muscle. (myocardial infarction). Serum myoglobin levels are measured by Electrophoresis.

4.

5.

E. PROTEINS AND NON-PROTEIN NITROGEN 1. What are the functions of proteins in normal physiology? Proteins function as transport carriers for other substances. They transport substances to the proper sites for absorption, modification, or other utilization. Salt fractionation The proteins are fractioned out by the use of salts. The salts decrease the water available for hydration of the hydrophilic groups and cause precipitation of the globulins. Zwitterion An ion that has both positive and negative regions of charge. Zeta potential This is the potential difference between the negative charges on the surface of the red blood cell membrane and the cations in the aqueous medium. Cations are divided into two groups, those that always move with the RBC and those that can move freely in the medium. The zeta potential is measured from the boundary of these two cations to the negative charge on the membrane. Page 15

2.

3.

4.

5.

Polypeptide Amino acids that combine to form proteins which link together to form peptides. Many peptides linked together form a polypeptide. Oligoclonal banding Electrophoretic pattern of CSF form patients with multiple sclerosis with distinct bands in the globulin zone. Briefly describe the Kjeldahl techniques for determination of protein and nonprotein nitrogen. In this method the serum proteins are precipitated with an organic acid. The nonprotein nitrogen is removed with the supernatent. The protein pellet is digested in H2SO4 with heat and a catalyst (cupric sulfate). Potassium sulfate can also be used to improve the digestion. The H2SO4 oxidizes the C, H, and S in protein into CO2, CO, H2O, and SO2. The nitrogen in the protein is then converted to ammonium bisulfite which is measured by adding alkali and distilling the ammonia into a standard boric acid solution. The ammonium borate formed is then titrated with a standard solution of HCL to determine the amount of nitrogen in the original protein solution. What are the major causes of increased and decreased albumin? Increased and decreased globulins?

6.

7.

8.

9.

What is the theory of refractometry? Refractometry- the velocity of light is changed as it passes the boundary between 2 transparent layers (air and water) causing light to be bent. When solute is added to water the refractive index at 20*C of 1.33 is increased by an amount proportional to the concentration of the solute in the solution. What are major interfering substances in the determination of serum protein by refractometry? Interfering substances- Nonprotien solids (electrolytes, urea, and glucose)

10.

11.

Name three ways to separate albumins from globulins. Electrophoresis, Chromatography Precipitation

Page 16

12. 13. 14.

Discuss the reasons for determining spinal fluid protein and glucose. What are normal values for spinal fluid protein and glucose? What results are expected from spinal fluid in meningitis ? In meningitis, encephalitis, and neurosyphilis there would be a decreased glucose level with increased protein levels (IgG). What results are expected from spinal fluid in encephalitis? In meningitis, encephalitis, and neurosyphilis there would be a decreased glucose level with increased protein levels (IgG). What results are expected from spinal fluid in neurosyphilis? In meningitis, encephalitis, and neurosyphilis there would be a decreased glucose level with increased protein levels (IgG). Discuss the BCG method for determining albumin? Why is the pH important? BCG (Bromocresol Green) method for determining albumin is a dye binding procedure where positively charged albumin is attracted to and binds to the anionic dye. Once bound to the albumin, the dye has a different absorption maximum than free dye. The amount of albumin can be quantitated by measuring the absorbance of the albumin-dye complex to which it is directly proportional. The pH must be adjusted on the solution to make the albumin positively charged so it will bind to the dye. What is Biuret reagent? Explain its function in determination of total protein. What are the major interfering substances? The biuret reagent contains sodium potassium tartrate to complex cupric ions to prevent their precipitation in the alkaline solution, and potassium iodide which acts as an antioxidant. In this procedure small peptides react and the color of the chelate produced has a different shade that seen with larger peptides (color varies from pink to reddish violet and is measured at 510nm). Major interfering substances are any compound with 2 or more of the following groups NHCH2 , and NHCS. What is a protein-free filtrate? List 3 precipitating reagents used. A protein free filtrate removes proteins from whole blood, serum, urine, or other body fluids by precipitation with a precipitant and then filtration or centrifugation. Some precipitating reagents are: o Tungstic acid o O-toluidine, o Horseradish peroxidase, o Molybdate Page 17

15.

16.

17.

18.

19.

Trichloroacetic acid

20.

What is Nessler's reagent? In what other reactions may it be used? Nessler's reagent: o Double iodide of mercury and potassium. o This reagent is used in the determination of non-protein nitrogens (NPN). o It is also used in Nessleration reactions of urea nitrogen. What is an A/G ratio? How is it used diagnostically? A/G ratio determines how much albumin is in the body compared to globulins. You use it in correlation with Total protein to determine if albumin or globulins are low.

21.

F. Specific serum proteins 1. What is alpha-1 antitrypsin? Ceruloplasmin? Name and describe disease processes involved with each protein. Alpha-1 antitrypsin o Acute phase reactant o Main function Neutralizes trypsin-like enzymes that can cause hydrolytic damage to structural protein. o Disease process: Severe deficiency of this is associated with severe degenerative emphysematous pulmonary disease. Ceruloplasmin o Copper containing glycoprotein that stores 90% of the total serum copper. o Disease process: Low serum concentrations of copper are associated with Wilsons disease. 3. What is the function of haptoglobin? Haptoglobin functions to bind free Hemoglobin by the alpha chain. Name three methods for measuring haptoglobin. 3 methods to measure Haptoglobin: o Starch gel electrophoresis, o Radial immunodiffusion, o Immunonephelometric methods.

2.

4.

When is haptoglobin decreased? Increased? Increased Haptoglobin levels: o Inflammatory conditions, Page 18

o Burns, and o Nephrotic syndrome Decreased Haptoglobin levels: o Intravascular hemolysis, o Transfusion reactions, o HDN, o Mechanical breakdown of RBC's, o Athletic trauma.

5.

Define Troponin Troponin- is a complex of 3 proteins that bind to the thin filaments of striated muscle (cardiac and skeletal) but are not present in smooth muscle. List three isoforms that make up the troponin complex. o 3 isoforms: TnT, TnI, TnC Describe the advantages of troponin over CKMB and name the method currently available to measure cTnI. o Cardiac troponin I is highly specific for myocardial tissue, and because it does not normally circulate in blood it is 13x more abundant in the myocardium than CKMB on a weight basis. o cTnI is very sensitive and can indicate even a minor amount of cardiac necrosis. o The relative increase of cTnI is greater than that of CKMB. o cTnI can be measured by: o Immunoenzymetric assays using 2 monoclonal Ab's directed against different epitopes on the protein.

6.

7.

G. Protein Electrophoresis 1. What is a monoclonal gammopathy? polyclonal gammopathy? A monoclonal gammopathy is a sharp narrow band in the late beta or gamma region that suggests a monoclonal M spike. The M spike is a spike of one class of Igs that is possibly metastasizing or producing clones of itself and suggests cancer, possibly multiple myeloma. A polyclonal gammopathy is a broad gamma band that is increased. It looks abnormal and is usually caused by an infection. It is made up of more than one serum protein being increased. Where are the sites of synthesis for the following proteins: a. albumin- liver b. alpha-1 globulin- liver c. alpha-2 globulin- liver Page 19

2.

d. beta globulin- liver e. gamma globulins- (IgA, IgG, and IgM)- made by B cells that become plasma cells. 3. Discuss the clinical picture of multiple myeloma. What are the expected results of electrophoretic patterns on serum and urine of a myeloma patient? In multiple myeloma you would see: o Bence jones proteins found in the urine (free kappa and lambda light chains) o "M" spike in electrophoretic pattern Briefly describe serum protein electrophoresis. Serum electrophoresis o Serum samples are applied to the cathode end of a support medium strip that is saturated with an alkaline buffer (8.6). The strip is connected to 2 electrodes and a current is passed. All serum proteins are negatively charged at the 8.6 pH so they migrate toward the anode end. Name the five bands that occur in serum and list the major proteins that migrate in the five bands. Diagram a normal pattern as they migrate from anode to cathode in a barbital buffer and label each peak.

4.

5.

6.

7.

What type of electrophoretic patterns can be expected in the following disease states? Explain the patterns and sketch them. a. Multiple myeloma

b. Nephrotic syndrome

c. Liver disease

d. Chronic infection

e. Acute phase reaction

f.

Malnutrition

Page 20

8.

Describe CSF protein electrophoresis. CSF protein electrophoresis is done with the same technique as serum electrophoresis, except agarose gel is used most often because it is a high resolution technique. What bands are normally seen in CSF protein electrophoresis Oligoclonal bands are: distinct bands seen in the globulin zone and they are associated with multiple sclerosis (90%), and Inflammatory infectious neurological disease. What are oligoclonal bands and what disease process are they associated with? pre-albumin, albumin, alpha-1 globulin (antitrypsin), insignificant alpha-2 globulin, Beta-1 zone (transferrin) Beta-2 zone.

9.

10.

11.

Describe protein electrophoresis patterns would appear in the following situations and explain why the pattern appears as it does.

a. Electrophoresis of plasma instead of serum

b. Serum containing alpha-feto protein

c. Fresh serum containing complement

d. Bisalbuminemia

e. Serum containing C-reactive protein

Page 21

H. Creatinine, Bun, Uric Acid

1.

What is NPN? Which compounds comprise 50% of the total NPN's? Non-protein nitrogen (NPN)- nitrogen containing compounds that remain in the blood sample after the removal of protein constituents. Urea is the compound that comprises 50% of the total NPNs.

2.

What is BUN? What is Azotemia? How is urea nitrogen converted to urea mathematically? Blood urea nitrogen (BUN)- nitrogen in the blood in the form of urea. This is a measurement that is used to analyze the urea level. Azotemia- an elevated level of urea in the blood. Bun x 2.14 = urea What is creatine? How is it measured? Creatine- a compound found in muscle synthesized from several amino acids. It combines with high energy phosphate to form creatine phosphate which functions as an energy compound. We can measure creatine by the Jaffe method which measures creatine based on analyzing the sample for creatinine before and after heating in acid solution. The heating converts the creatine to creatinine and the difference between the two samples is the creatine concentration.

3.

4.

Relate elevations in uric acid to the following disease states:

Primary gout- this is caused by increases of uric acid which cause sodium urates to precipitate in the joints. This can be caused by overproduction of uric acid, drugs, and alcoholism. Secondary gout- this gout is formed as a secondary infection caused by a larger problem like leukemia. Leukemia- this causes the increased breakdown of cell nuclei caused by chemotherapy which causes the uric acid levels to increase. Polycythemia- this causes the increased breakdown of cell nuclei caused by chemotherapy, much like leukemia does, which also increases the uric acid levels to increase. Glomerular nephritis- in this disease the nephrons of the glomerulus are damaged in the kidney which causes poor filtration and increased levels of the uric acid.

Page 22

Multiple myeloma- this disease is also treated with chemotherapy which breaks down the nuceli and causes the uric acid to increase much like leukemia and polycythemia.

5.

Relate uric acid production to purine catabolism. Uric acid is formed from the catabolism of purines like adenosine and guanine in the liver. This uric acid is transported by the plasma from the liver to the kidney where it is filtered by the glomerulus into the proximal tubules where most is reabsorbed and only small amounts are secreted into the urine. What is the clinical significance of BUN? (renal, prerenal, postrenal). What creatinine values are expected in these conditions? BUN- (direct urea measurement from serum or plasma) It is used extensively in the determination of renal function. BUN CREATINE BUN/CR Increased Normal Decreased

6.

RATIO Pre-renalRenalPost renal7.

Increased Normal Increased Increased Normal Increased

What are normal values for BUN and creatinine? What is the normal ratio of BUN to creatinine? When is the ratio altered? Normal BUN- 7-18mg/dl Normal Creatinine- 0.5-1.2mg/dl Normal BUN/Creatinine ratio- 10:1-20:1 The ratio is altered in: o Low protein uptake o Acute tubular necrosis o Severe liver disease Diagram and describe the Berthelot reaction for BUN. Berthelot Reaction for BUNo Urea is hydrolyzed with urease, and the ammonia ion formed is reacted with phenol and hypochlorite in alkaline medium to form indophenol. Nitroprusside is used to catalyze the reaction. o Absorbance of dissociated indophenol (blue chromogen) is measured at 560nm. o REACTION: NH4 + 5NaOCC+ 2 phenol indophenol + 5NaCl + 5H2O What is the purpose of the following reagents? o urease- used ot prepare the stock suspension Page 23

8.

o o o 9.

sodium nitroprusside- catalyzes the reaction phenol- converted to indophenol alkaline hypochloride- aids in conversion of phenol to indophenol.

What are some advantages and disadvantages of this method? What anticoagulant must be avoided when using any urease method? o Advantages- can use serum, plasma, or urine. o Disadvantages- contamination of urine with bacteria is common and can cause decreased urea and formation of ammonia. What kidney functions do the following clearance tests measure? inulin- Reference substance for measuring GFR (glomerular filtration rate) creatinine- universely used in assessment of GFR. urea- not useful in monitoring GFR, but serum urea may provide useful clinical metabolic information. p-amino-hippurate- reference substance for the measurement of renal plasma flow.

10.

11.

Diagram and describe the Jaffe reaction for creatinine. What substances give false positive reaction? Jaffe reactiono The reaction occurs between creatinine and the picrate ion formed in the alkaline medium and a red-orange adduct develops. Teh observed rate of the hydroxyl ion concentrations over a broad range of picric acid concentrations. This is measured spectrophotometricaly at wavelengths of 485-520nm. Substances that give false positives are: Protein, Glucose, Vitamin C (ascorbic acid), Acetone, cephalosporin Why is creatinine preferred to urea for clearance tests? What data are necessary to calculate creatinine clearance? Write the formula. What are the normal values for creatinine clearance? Creatinine is more specific for kidney function than urea is. Data necessary for calculating creatinine clearance: o Urine volume, o Creatinine concentration. in urine, o Creatinine concentration in plasma Creatine Clearance = (Urine Cr x Urine volume)/ (Plasma Cr) Normals: Male= 97-137 ml/min Females = 88-128 ml/min

12.

Page 24

13.

A creatinine clearance was performed on a male patient 1.5m tall and weighing 65kg. His blood contained 2.5 mg/dl creatinine. The urine creatinine was 50 mg/dl and the urine volume was 300 ml/4hrs. What was the creatinine clearance for this man? Creatine Clearance = (50 x 300)/ 2.5 x 1.76/ 1.60 (body surface area *see chart) What is creatinine? What is the normal range? What single disease state is associated with elevated creatinine. Creatinine is a compound formed when creatine or creatine phosphate spontaneously loses water or phosphoric acid. It is excreted into the plasma at a relatively constant rate in a given individual and excreted in the urine. Its decrease is associated with renal dysfunction as in glomerulonephritis. Diagram the reaction, list reagents used and describe the principle of the oxidation reduction method for uric acid using phosphotungstic acid. This is the most common method used. It is based on the oxidation of uric acid in a protein-free filtrate with subsequent reduction of phosphotungstic acid to tungsten blue. It uses Na carbonate to provide the alkaline pH necessary for the color development. The blue color produced can be intensified by adding cyanide or by keeping the proper pH.

14.

15.

I.

Miscellaneous Proteins 1. Describe the method for detecting phenolketonuria? What enzyme deficiency results in phenolketonuria? Phenolketonuria results from a total absence of or absence of activity of the enzyme phenylalanine hydrolase (AKA phenylalanine-4-mono-oxygenase). You can use the Guthrie test to detect PKU.

2.

What are cryoglobulins? How are they measured? Cryoglobulins are serum protein that precipitates at temperatures lower than body temperature. What is alpha-fetoprotein? What does its presence signify? How is it detected? Alpha-fetoprotein is a globulin protein synthesized in the fetal yolk sac and then by the parenchymal cells of the liver.

3.

Page 25

It is measured to determine if there is increased passage of fetal proteins into the amniotic fluid. We also measure the alpha-fetoprotein levels in association with spinabifiida, renal tube defects, and general fetal distress.

4.

What is carcinoembryonic antigen? What types of tumors is it most frequently associated with? What types of methods are used to measure it? Carcinoembryonic anitgens (CEA) are glycoproteins which are associated with numerous cancers (colon, lung, pancreas, stomach, or breast tissue tumors) For each of the following tumor markers describe the types of tumors they are most often associated with and how they are measured. CA125- ovarian cancer. PSA- (prostate specific antigen)- Prostate cancer

5.

J. ENZYMES

1.

(L) Give the substrates for the following: LD, CK, AST, ALT, GGT, CK-MB LD Lactate o Catalyzes oxidation of Lactate to Pyruvate and the reverse reaction of Pyruvate to Lactate o ischemia, o myocarditis, o cardiac congestion; CK Creatinine o Catalyzes the reversible phosphorylation of ATP o Muscular dystrophy, o muscle malignancies, o heart disease, o thyroid disease, o CNS disease. AST Aspartate o Transfers amino acids o This is higher in neonates due to their immature liver o Liver disease (20-100 times normal in hepatitis), o carcinoma, o cirrhosis, o liver disease o heart disease, o muscle disease, o gallbladder disease, o AMI, o pulmonary embolism Page 26

ALT o o o o o o o GGT o

Alanine Catalyzes the transfer of an amino group of alanine to alpha-ketoglutarate Enzymatic-UV Monitoring Liver disease, carcinoma gallbladder disease, cirrhosis, hepatotoxicity Glutathione Transfers gamma-glutamyl residue from gamma-glutamyl peptides to amino acids, water, and other small peptides o Liver disease, o obstructions of the internal liver or gallbladder, o alcoholism, o pancreatic problems CK-MB Creatinine o Catalyzes the reversible phosphorylation of ATP o Myocardial problems

2.

(L) Define: a. Isoenzymeo one of several forms in which an enzyme can exist in various tissues. o Although they are similar they can be separated from each other by special chemical tests (electrophoresis) to give more specific information. b. Coenzymeo These are enzyme activators that are usually heat stable and of low molecular weight. o When these are combined with an inactive protein called an apoenzyme they form an active compound or a complete enzyme called holoenzyme. c. Catalysto Substance that speeds up the rate of a chemical reaction without itself being permanently altered of used up in the reaction. o They are effective in small quantities and are not used up in the reaction. They can be recovered unchanged. d. Activator o Substance in the body that converts an inactive substance into an active agent. o Example: the hydrogen ions on pepsinogen converting it to pepsin. e. Inhibitoro Chemical substance that stops the enzyme activity. f. Hydrolaseo Enzyme that causes hydrolysis. These catalyze bond cleavage by the addition of water. g. OxidoreductasePage 27

h. Enzyme that removes electorns and their corresponding electrons. i. Transferaseo These enzymes move chemical grouping from one compound to another. 3. Relate amylase and lipase activity to the following disease states: a. Acute pancreatitis- both increase in this. b. Malabsorption- increased in both. c. Chronic pancreatitis- increased in both d. Pancreatic carcinoma- increased in both e. Cystic fibrosis- increased in both because it leads to malabsorption What are the sources of acid phosphate in the body? What are normal ranges for acid phosphatase in males and females? o Acid phosphate is found in most tissues in the body like bone, bone marrow, liver, spleen, RBCs, platelets, and in the highest concentration in the prostate gland of the male. 8. (L) Explain the clinical significance of alkaline phosphatase in the following disease states:

4.

5.

a. b. c. d. e. 6.

Obstructive jaundice- increased ALP levels Parenchymal jaundice- increased ALP levels Paget's disease- increased ALP levels Hyperparathyroidism- increased ALP levels Pregnancy- increased ALP levels

List enzymes elevated in hemolysis? What enzyme might be depressed with refrigeration and freezing? Enzymes elevated in hemolysis: o CK, LD, AST (aspartate transferase), ACP (acid phosphatase), ALP (alkaline phosphatase), and LIPASE Enzymes depressed by refrigeration: o LD (occurs at 4*C within 24 hours), and LIPASE (occurs if stored at 4*C for 3 weeks) What is cholinesterase? Why is it important in presurgery cases? Cholinesterase: an enzyme found in RBCs, lungs, spleen, nerve endings, and brain. It is responsible for the prompt hydrolysis of acetylcholine released at the nerve endings to mediate transmission of the neural impulse across the synapse. The degradation of acetylcholine is necessary to the depolarization of the nerve so that it can be repolarized in the next conduction. o This is important to measure cholinesterase in presurgery to determine the amount of succinyl dicholine (muscle relaxer) that can be given in surgery without complications. (You can only give the amount of succinyl dicholine to the patient that the patients cholinesterase can rid their body of.) Page 28

7.

8.

What is the relationship of amylase and lipase in pancreatic disease? Why are both tests necessary in the monitoring of the disease? o Amylase and Lipase are both elevated in pancreatic disease. o You must monitor both amylase and lipase in pancreatic disease because: Amylase is more sensitive, but less specific (b/c also found in other parts of the body) Lipase is more specific, but found in small quantities (less sensitive b/c only found in the pancreas) Discuss the following in relation to amylase activity: activators, pH, temperature. o Activatiors: Calcium and Chloride ions o pH- optimal is 6.9 - 7.0 o Temperature- optimal is 37-40*C Discuss prostatic disease and acid phosphatase levels in serum. Total activities of ACP may reach 40-50 times the normal in severe stages of prostate cancer. If the carcinoma is highly localized to the prostate there may only be slight increases in ACP activity. In benign hypertrophy of prostate, enzyme levels are normal. How is the L(+) tartrate utilized in the determination of acid phosphatase? Tartrate inhibits the activity of non-prostatic ACP so that specificity is enhanced when it is used. Explain heat separation of alkaline phosphate isoenzymes. How does heat effect the liver fraction? bone? placenta? Heat separation of alkaline phosphatase Isoenzymes ALP activity is determined by measuring ALP before and after heating serum at 56*C for 10 minutes. Placental ALP is the most heat stable followed by intestinal, liver, then bone. Placental ALP will resist heat denaturation at 65*C for 30 min. If the residual activity after heating is <20% of the total prior to heating then it is bone phosphatase. If the residual activity after heating is >20% of the total prior to heating then it is Liver phosphatase.

9.

10.

11.

12.

K. Enzyme Electrophoresis

1.

List the CK isoenzymes. Describe the makeup of each fraction and organs associated with each fraction. How are isoenzymes separated? CK-1 ( brain, brain subunits)- brain, prostate, uterus, bladdar, placenta CK-2 (muscle, brain subunits)- heart muscle and skeletal muscle CK-3 (muscle, muscle subunits)- sketal muscle and heart muscle Page 29

2.

How can hemolysis affect the LDH electrophoretic pattern? What is the clinical significance of this? Hemolysis can cause LDH electrophoresis to have an LD-1 to LD-2 flip. Using a hemolyzed sample would cause the results to have a LD-1 to LD-2 flip as seen in cases of Myocardial Infarctions and Hemolytic anemia.

L. ELECTROLYTES AND TRACE ELEMENTS 1. For each of the following give normal range, panic values, categorized as anion or cation if applicable and categorize as intracellular or extracellular if applicable. Potassium Cation Sodium Cation Calcium Cation Magnesium Cation 2. Bicarbonate (HCO 3) Anion Chloride (Cl) - Anion
-

Describe the relationship between electrolyte balance and water balance. Include the roles of the kidney, hypothalamus, ADH and the renin-aldosterone system. The electrolyte balance and water balance are directly related. The plasma sodium concentration depends greatly on the intake and excretion of water. If the sodium increases it stimulates thirst which will increase the intake of water, and the kidneys have the ability to conserve or excrete large amounts of sodium depending on the blood volume which is directly related to the water volume. The excretion of water is largely affected by the ADH (which is secreted from the hypothalamus) release in response to the increase in blood volume. The renin-aldosterone system acts in the kidney to increase the retention of sodium and increase the excretion of potassium which will eventually increase the blood pressure by using the electrolyte to balance the water level. List anticoagulants of choice and the effect of hemolysis if any on the following ions: K, Na, Ca, Mg, HCO3, Cl, P04, Fe. All electrolytes should be determined using serum or heparinized plasma. o K- hemolysis increases K levels o Mg- hemolysis increases Mg levels o HCO3- hemolysis increases HCO3 levels o P04- hemolysis increases PO4 levels o Fe- hemolysis increases Fe levels o Na-hemolysis does not effect this significantly Page 30

3.

o o

Ca- hemolysis does not effect this significantly Cl- hemolysis does not effect this significantly

4.

Identify the major functions of sodium, chloride, bicarbonate, and potassium. Sodium- this electrolyte largely determines the osmolality of the plasma. Chloride- this electrolyte maintains the electrical balance by balancing the sodium charge, and using the chloride shift with bicarbonate. Bicarbonate-this electrolyte is used to maintain the acid base balance and buffer the blood. Potassium- this electrolyte regulates the neuromuscular excitability, contraction of the heart, ICF volume, and hydrogen ion concentration (pH). Define and list conditions associated with each of the following: Hypernatremia- elevated levels of sodium in the blood and is associated with increased sodium concentration because of excess water loss. It can be caused by increased sodium intake or decreased water intake. Hypokalemia- decreased levels of potassium in the blood and is associated with GI or urinary loss of potassium , or with increased cellular use of potassium. This can be caused by vomiting, diarrhea, etc. Hyperkalemia- increased levels of potassium in the blood and is associated with diabetes mellitus, or metabolic acidosis.

5.

Hyponatremia- this is decreased levels of sodium in the blood and is associated with the blood volume status. It results from sodium loss in excess of water loss.

6.

What disease process results with increased bicarbonate? decreased bicarbonate? Metabolic acidosis is related to the decrease in bicarbonate. Metabolic alkalosis is related to the increase in bicarbonate. Define titration. Explain its application to chloride methods. Titration is the diluting out of a sample with a liquid reagent of a known strength and measuring the volume necessary to convert the sample through a given reaction. In the chloride titration method the chloride ions combine with the mercuric ions to form soluble and undissociated mercuric chloride. The proteins in the serum are precipitated with the tungstic acid and an aliquot of the filtrate is titrated with an acidic solution of mercuric nitrate using a color indicator. This color indicator turns violet-blue at the first excess of mercuric ion. Page 31

7.

8.

List disease processes in which hyperchloremia and hypochloremia occur but sodium is normal. Hyperchloremia- this is an increase of serum chloride and occurs in situations where there is an excess loss of bicarbonate ion due to GI losses, RTA, or metabolic acidosis. Hypochloremia-this is an decrease of serum chloride and occurs with the excess loss of chloride from prolonged vomiting, diabetic ketoacidosis, or aldosterone deficiency. List causes of an increased anion gap? decreased anion gap? Decreased anion gap is rare, but may be seen in multiple meyloma because of abnormal proteins. It can also be caused by instrument error. Increased anion gap- this may be caused by uremia, ketoacidosis (seen in starvation or diabetes), posioning due to ingestion of substances like methanol or ethylene glycol, lactic acidosis, or severe dehydration which causes increased plasma proteins or instrument error. Explain why stock standards of sodium and potassium are kept in plastic containers. Stock standards of Na and K are kept in plastic containers because glass containers leach out the Na and K from the sample. What is an "anion gap". List formulas for its determination. Anion gap- the difference between unmeasured anions and unmeasured cations. It is useful for indicating an increase in one or more of the unmeasured anions in serum. AG = Na - (Cl + HCO3) AG = (Na + K) - (Cl + HCO3) Why is an anion gap routinely performed on all sets of electrolytes? What is an unacceptable gap? What is standard operating procedure when an anion gap is unacceptable? o An anion gap is routinely performed on all sets of electrolytes because it is useful in indicating an increase in one or more of the unmeasured anions in serum, and for QC on th analyzer (an abnormal gap can indicate an analyzer problem if performed on a person in good health). o An acceptable gap is 10-20:1, so greater than or less than that would be considered unacceptable and would need to be rerun.

9.

10.

11.

12.

13.

Describe the impact of each of the following on serum potassium levels. administration of insulin- decrease serum K levels (increases the cellular uptake of K) Page 32

14.

acidosis- increase serum K levels (excess H enters cell to be buffered and causes K to leave the cell to maintain electro neutrality) alkalosis- decreases the serum K (increases the cellular uptake of K)

Discuss the following factors influencing serum calcium and phosphorus levels: parathyroid hormone- this hormone is used to increase the absorption of calcium and increase the excretion of phosphorus. To increase calcium it breaks down the bone to release Ca (bone resprption), it conserves Ca by increasing the tubular reabsorption in the kidney and it stimulates the renal production of vitamin D which also increases the Ca absorption. To decrease phosphorus the blood concentration the PTH increases the renal excretion. calcitonin- this hormone is used to decrease calcium levels and increase the phosphorus levels which inhibits the actions of PTH and vitamin D. Vitamin D (calcitriol)-when the calcium is decreased or the phosphorus is increased this hormone is used to increase calcium by aiding the effects of PTH by causing more calcium to be stored or released. It decreases phosphorus by increasing the absorption of it in the intestines and increasing the reabsroption in the kidneys. plasma proteins-Albumin is the plasma protein that maintains the appropriate fluid in the tissues, and it binds various substances in the blood like calcium. serum pH- a decrease in pH will increase the phosphate levels in the serum like seen with antiacids. Discuss calcium, phosphorus and PTH levels related to the following disease states: bone disease-calcium will be normal to low, phosphorus will be normal to low, and PTH will be normal to high. malabsorption- calcium will be decreased, Phosphorus will be decreased, and PTH will be increased. renal failure- calcium will be low to normal, phosphorus will by high, and PTH will also be high. liver disease- calcium will be decreased, Phosphorus will be decreased, and PTH will be increased. primary hyperparathyroidism-Calcium will be Increased, Phosphorus will be decreased, and PTH will be high. secondary hyperparathyroidism-Calcium will be decreased, Phosphorus will be low to normal to high, and PTH will be increased. primary hypoparathyroidism-Calcium will be decreased, Phosphorus will be increased, and PTH will be decreased. secondary hypoparathyroidism-Calcium will be decreased, Phosphorus will be increased or decreased, and PTH will be decreased.

15.

Page 33

16.

What is the physiologic relationship between calcium and phosphorus? Why? What are the physiologic functions of calcium and phosphorus? Calcium and phosphorus are inversely related in the serum because phosphate is an intracellular anion, and calcium is an extracellular cation.. Calcium functions in bone matrix, as an enzyme activator, in coagulation and complement, and in muscle contraction. Phosphorus functions in production of ATP, GTP, CTP, UTP, and DNA structures, as a major body buffer, and in the bone matrix. What is the relationship of calcium levels to alkaline phosphatase activity? Alkaline phosphatase levels are increased in periods of bone growth or reconstruction which uses up the calcium and causes the serum levels of calcium to decrease. They both also function in bone matrix.

17.

18.

Describe three forms of body calcium. To which form is PTH most sensitive? How can only "active" calcium be measured? Calcium forms- free-ionized Ca, protein bound calcium, and as complexed salts. PTH is most sensitive to ionized calcium. To measure only the active calcium you must measure it under anaerobic conditions because an increase in pH can cause the protein bound Ca to increase which decreases the ionized Ca, and decreasing the pH can cause the protein bound Ca to decrease and the ionized Ca to increase. Define and describe tetany? What are the relationship of magnesium and calcium to tetany. Tetany- irregular muscle spasms. Calcium- a rapid decrease in ionized calcium concentration will cause tetany. Magnesium- this is required along with ATPase for normal Ca uptake following a contration. It is also required for muscle cell stimulation by regulating the acetylcholine which is a potent neurotransmitter. Describe the relationship between parathyroid hormone and magnesium levels. The parathyroid hormone increases the renal absorption of magnesium and enhanses the absorption of magnesium in the intestine. PTH regulates Ca, Na, and Mg.

19.

20.

21.

What is the physiologic role of magnesium? Where is magnesium stored in the body? o Magnesium functions as a cofactor for more than 300 enzymes including those important in glycolosis, transcellular ion transport, neuromuscular transmission, synthesis of carbohydrates and many others. o Magnesium is stored in the bone (53%) and the rest (46%) is in the muscle, soft tissue and other organs. Page 34

22.

Discuss the following as they pertain to magnesium: o alcoholism- people who are alcoholics tend to have diets deficient in magnesium or have problems with malabsorption (hypomagnesemia). o malabsorption- this causes a decrease in magnesium because it is not absorbed (hypomagnesemia). o magnesium sulfate therapy- this is given parenterally to severely ill patients. o secondary hypoparathyroidism- this may cause an increased renal excretion of magnesium due to an excess of calcium ions. What is the physiologic function of iron? How is iron transported in the body? How is iron stored in the body? Iron functions as part of heme in hemoglobin. It is transported by transferrin and stored in the body as ferratin and hemosidrin. Describe iron levels, %Saturation, TIBC, and ferratin levels in the following disease states: iron deficiency anemia-the % saturation is decreased, TIBC is increased, and ferratin is decreased. anemia of chronic infection- the % saturation is normal, TIBC is decreased, and ferratin is normal to increased. hemochromatosis- the % saturation is increased, TIBC is increased, and ferratin is increased. What is the physiologic function of copper? What is Wilson's disease? ceruloplasmin? Copper functions as an enzyme indicator, it acts on ferroxidase, and it acts it is a component of enzymes or proteins involved in redox reactions. Wilsons disease- also known as hepatolenticular degeneration is a genetically determined defect in ATPase where the copper is transported normally from the intestine into the liver, but cannot be transported from the liver into the bile. What is the physiologic function of zinc? How is it measured? Zinc functions as a cofactor for more than 300 enzymes. It can be measured by atomic absorption. Define: total iron binding capacity- (amount of transferrin bound already) An estimate of serum transferrin levels obtained by measuring the total iron binding capability of a patients serum. Since transferrin represents most of the iron binding capacity of serum TIBC it is generally a good estimate of serum transferrin levels. % saturation- (transferrin saturation in the patients sample) % saturation = total iron/TIBC x 100

23.

24.

25.

26.

27.

Page 35

28.

unsaturated iron binding capacity- (the amount of sites available to bind iron in patients sample) UIBC = iron added excess iron Serum iron = TIBC UIBC latent iron binding capacity- (estimate of non-reacting iron bound to transferrin)

What is the source of blood ammonia? How is it detoxified? What disease processes are associated with increased ammonia? How is ammonia related to Reye's Syndrome? Blood ammonia arises or comes from the deamination of amina acids through the action of degestive and bacterial enzymes on proteins in the intestinal tract. Ammonia is released from metabolic reactions that occur in skeletal muscles during excercize. Ammonia is detoxified by the liver in the urea cycle where ammonia is converted to urea. Hepatic failure, Reyes syndrome, and urea cycle enzyme deficiencies are associated with increased ammonia. Reyes syndrome is usually following a viral infection and it uses ammonia levels to correlate with the severity of the disease and prognosis.

29. What is the purpose of renal synthesis of ammonia? The kidney synthesizes ammonia to compensate for metabolic acidosis. 30. What are the three ketone bodies. When are they formed? List disease processes associated with increased ketones. o acetone, acetoacetic acid, and beta-hydroxybutyric acid. o These are formed as a product of incomplete fat metabolism, and are associated with diabetes mellitus, starvation, and prolonged vomiting. What is lactic acidosis? When does it occur? o Lactic acidosis- an increase or accumulation of lactic acid in the blood. o This occurs if there is improper oxidation of skeletal muscle and other tissues. What role does hemoglobin play as an important buffer system in the body? What is carbonic anhydrase? What is chloride shift? Hemoglobin buffers the blood by delivering oxygen to the tissues and then taking the carbon dioxide to the lungs to be exhaled. Carbonic anhydrase- this enzyme catalyzes the reaction of carbon dioxide to bicarbonate and Hydrogen ion. The chloride shift- the carbon dioxide goes into the RBC and forms carbonic acid, this acid splits into hydrogen ion and bicarbonate. The bicarbonate leaves the cell and makes it more negative outside the cell, and more positive on the inside because of the hydrogen ion. At this point the chloride shifts into the cell to balance the electorneutrality of the cell.

31.

32.

Page 36

33.

What are the main factors which influence the oxygen binding ability of hemoglobin? Specifically how do acidosis and alkalosis affect O2 saturation? If there is increased oxygen then the hemoglobin binds more oxygen and if there is less oxygen the hemoglobin picks up less oxygen and wants to hold on to it. The ability for hemoglobin to bind oxygen depends mostly on the availability of oxygen. Acidosis- the pH drops drastically and increases the hemoglobin affinity for O2. Alkalosis- the pH is increased and it decreases the hemoglobin affinity for O2. Why is heparin the anti-coagulant of choice for pH and blood gas work? How does it work? Heparin is used because it holds the ph constant in blood and prevents the change in gals levels in the sample.

34.

35. Why are blood gas specimens placed in ice immediately after collection? Blood gas specimens are placed on ice immediately after collection because: The pH decreases with time if it is not placed on ice immediately after drawn. The lower temperatures prevent the cells from undergoing glycolysis. IX. ENDOCRINOLOGY

36.

Give expected T4, T3 uptake, FTI and TSH levels in the following:

37.

What is the function of the thyroid hormones in body metabolism? Describe the following disease processes: The thyroid hormone function is to stimulate the metabolism. Without the thyroid hormone the metabolism will decrease and the patient will experience obesity, mental retardation, edema (water in the tissues), decreased body temperature, and anemia. Page 37

cretinism- condition where the patient has a dysfunctional or no thyroid causing mental retardation and death. Autosomal recessive. juvenile myxedema-A dysfunction of the thyroid after birth that has very severe consequences. All 50 states require a T4 screening test for neonates. adult myxedema- A dysfunction of the thyroid later in life (>30 years) causing hair loss, dry skin, yellow pallor, thick tongue, and arterosclerosos. endemic goiter- an enlarged thyroid due to lack of iodine. hashimoto's disease- the most common form of thyroid disease occurring mostly in women 40to 60 years old. Treat with thyroxine. Autoimmune disease. grave's disease- the patient has and AB to TSH receptor on the thyroid gland. This AB causes the thyroid gland to think it is TSH and start to release T3 and T4 even though there is a decrease in actual TSH. thyroid tumors (goiters)- Tumors that cause the dysfunction of the thyroid gland allowing abnormal amounts of hormone to be released.

38.

What are the roles of TSH? What amino acids are necessary for synthesis? Body temperature stimulates the thyroid hormone which stimulates the hypothalamus and causes it to release TRH to the pituitary gland. The TRH causes the pituitary gland to release TSH to stimulate the thyroid gland which then releases T3 and T4. TheT3 and T4 cause the body temperature to rise and act as the shunt to tell the hypothalamus to stop making the TRH and the pituitary to stop making TSH. What is thyroglobulin? thyroid binding globulin? Thyroglobulin- an iodine contaning protein secreted by the thyroid gland. Thyroid binding globulin- A protein that binds thyroid hormones. The TBG assay is used to confirm results of T3 and T4, or abnormalities it the relationship of the TT4 and T3U test. They can also be used as tumor markers for thyroid cancer. Briefly describe each of the following groups synthesized by the adrenal cortex. What molecule is necessary for their synthesis? glucocorticoids- (cortisol) these stimulate gluconeogenesis to increase glucose, encourage the glycogen production and release from the liver, block epinephrine, act as and anti-inflammatory, inhibit WBC migration, phagocytosis, increase hematopoiesis and stabilize lysozome. mineralocorticoids- (aldosterone) these regulate potassium metabolism and regulate extracellular fluid volume. They also control water secretion by regulating Na absorption and K secretion. androgens- (testosterone, dehydroepiandrosterone, dehydroepiandosterone sulfate) These function in spermatogenesis and in the formation of the secondary male sex hormones. estrogens- (estrogen, estradiol, estriol) these function in ovulation, embryo preparation for implantation, and menstruation regulation Page 38

39.

40.

41.

Describe the feedback mechanism for cortisol. What is the function of cortisol? What is the role of ACTH? The hypothalamus is stimulated by low glucose to secrete CRH which causes the anterior pituitary to secrete ACTH to help make 11-deoxycortisol. Cortisol is formed and is found bound to transcortin or as free cortisol. The free cortisol stimulates he glucose production and when the need is met it acts as a shunt to tell the hypothalamus to stop making CRH. Diagram the renin-angiotensin-aldosterone system. What is the purpose of this system? The afferent arterioles secrete rennin when there is a decrease in blood pressure. The rennin converts angiotensinogen to Angiotensin I which is converted to Angiotensin II. Angiotensin II stimulates aldosterone to be produced and secreted to increase the blood pressure by retaining sodium to passively hold water and increase the blood volume which increases the blood pressure. What hormones make up the 17-hydroxysteriods? the 17-ketosteroids? Why are these tests performed? Cortisone, cortisol, and 11-dehydroxycorticoids make up 17-hydroxysteroids. Androgens make up the 17-keytosetriods. These tests are performed to observe the levels for glucosteriods, testosterone, and other androgens. What are the catecholamines? Where are they synthesized? What is the precursor amino acid? What is their basic function? Catecholamines are epinephrine, norepinephrine, and dopamine. They are synthesized in the adrenal medulla of the adrenal glands. The precursor amino acid is tyrosine. Their basic function is to increase the blood pressure and heart rate when the body is excited in the fight or flight mechanism. They have the ability to break down adipose tissue and glycogen to be used for energy in the fight or flight mechanism. Describe catecholamine levels in the following diseases: Pheochromocytomas-Tumors of the adrenal medulla or the sympathetic ganglia of the adrenal glands which cause overproduction of the catecholamines. Infantile Neuroblastoma- Tumors of the adrenal medulla that cause an increase in catecholamines. What is the function of prolactin? When is it measured? Prolactin acts to initiate lactation and maintain it in a pregnant female. It is measured to evaluate pituitary tumors, galactorrhea, and fertility. Describe levels of FSH and LH during follicular development, ovulation and the luteal phase? Why and how are these hormones measured? FSH increases during follicular phase, goes down and rises again just after ovulation, and drops slowly toward luteal phase.

42.

43.

44.

45.

46.

47.

Page 39

LH is baseline in follicular phase, increases in ovulation, and decreases back to baseline during the luteal phase. These hormones are measured by RIA to assess pituitary and gonadal axis, fertility problems, to investigate puberty problems (late puberty), and to investigate pituitary tumors.

48.

What is the function of gastrin? How is it related to Zollinger Ellison Syndrome? Gastrin functions to secrete HCl to digest food, pepsin, intrinsic factor, pancreatic hormones, and bile from the liver. Zollinger Ellison (ulcers in the stomach or duodenum) syndrome has increased Gastrin levels, so it can be separated from other peptic ulcers because they dont increase in production of gastrin. What is the function of the androgens? How are they measured? How is testosterone measured? Androgens function in spermatogenesis and formation of secondary sex characteristics of males. Androgens like testosterone are measured by RIA. Briefly describe the function of the estrogens and progesterone. What test is used for total urinary estrogens? What is its principle? Estrogen and progesterone both function in getting the uterus ready for embryo implantation. The Brown method (a colorimetric method using Kober reagent to look for a pink color indicating that estriol is present) is used to measure total estrogen levels. What is the primary site of synthesis of estradiol, and esterone? Explain why these two estrogens are measured? Estradiol is produced in the ovary of a pre-menopausal woman. Esterone is produced in the adrenal cortex of the adrenal glands of a postmenopausal woman. Both are measured to determine menstrual cycle disorders and ovulation in women. What does the T3 uptake measure? Why is this important? T3 uptake measures- the # of free binding sites in the Thyroxine Binding Globulin. This is important because it indirectly gives the amount of thyroxine in the serum sample. What is the free thyroxine index? T7? T12? How are they calculated? What is the importance of this calculation. Free thyroxine index- estimates how much free T4 is in the blood. T7 and T12- are pseudonyms for FTI (T7 = T3+ T4, and T12 = T3 x T4) FTI- an index of thyroid status, it provides T4 and T3U which are useful in diagnosing thyroid problems. Free thyroxine index = (T4) x (% T3 uptake as a decimal) Page 40

49.

50.

51.

52.

53.

54.

Discuss the purpose of each of the following steps in hormone determinations: hydrolysis- remove and solubilize attachments (chemically or enzymatically) purification- organic solvents purify the hormones extraction- centrifugation, washing, and ion exchange estimation- react, detect, and quantitate by various methods What is diurnal variation? How does this impact cortisol levels and the collection of cortisol specimens? Diurnal variation- levels of analytes rise and fall, they peak early in the day when most of us are asleep. Cortisol levels and ACTH (anterior pitutitary hormone, Cortocotropin) They rise between 0600 and 0800 hours, then decrease all day long. At 2000 hrs (8 pm), the level is 2/3 of what it is at 0800 hrs, so the best analysis is made form early morning specimens. Describe Cushing's Syndrome and Addison's disease. Include their impact on diurnal variation, cortisol levels, glucose levels, aldosterone levels, electrolyte and water balance. How may secondary disease states be diagnosed? Cushinns syndrome- Increased- cortisol, diurnal variation, aldosterone, hypertension, and hyperglycemia (glucose), and decreased potassium. Addisons disease- Decreased- cortisol, diurnal variation, aldoserone, hypotension, and hypoglycemia (glucose). Increased- potassium. Addisons disease- Cosyntropin is given to th epatient which causes the cortisol release, which aids in determining if patient has Addisons disease, because you look to see if cotrisol is released by the cortisol stimulating drug. If still no release of cortisol, it is a primary disease like addisons. Cushings disease- use hiht dexamethasome suppression test where there is a suppression of urine and plasma cortisol which only occurs in cushings disease. When are plasma epinephrine and norepinephrine levels useful? Epinephrine and Norepinephrine levels are useful in diagnosing stress, increased catacholamines, decreased blood pressure, decreased blood volume, thyroid hormone deficiency, and congestive heart failure. Decreased catacholimines are found in hypotension. When is it valuable to quantify HCG? What trimester of pregnancy are HCG levels used to monitor fetal health? It is valuable to quantitate HCG to indicate how far along in pregnancy a patient is, or if pregnant at all. Fetal health can be determined in the 1st and 3rd trimesters by looking at BHCG levels. What is the function of serotonin? When does it increase? What is 5-hydroxy indolacetic acid? How is 5-HIAA measured? Serotonin is released during coagulation by platelets and is involved in smooth muscle stimulation and vasoconstriction. It is increased with carcinoid tumors that Page 41

55.

56.

57.

58.

59.

occur in the Ileum and appendix, 5-HIAA is a metabolite of serotonin and is excreted in the urine. It is measured colorimetrically after reacting it with 1-nitroso2-napthol and nitrois acid (purple color). 60. Why are estriol levels measured? What trimester of pregnancy are these levels useful in? Estriol levels are measured to help determine pregnancies. Levels in non pregnant females can be measured in the ugs, but pregnant women have levels in the mg range. These levels are useful in the third trimester (last 4-6 weeks of pregnancy)

M. TOXICOLOGY 1. List methods in which urines are screened for drugs of abuse? Why is urine preferred? What is the purpose of extraction? You can use thin layer chromatography, gas chromatography, and immunoassays to screen for drugs of abuse. Urine is used most often because the drugs are filtered through the kidneys and show up in the urine. Why are chloramphenicol levels monitored? What disease process may result from chloramphenicol. Chloramphenicol levels are monitored to make sure that ALA synthase is being produced in enough quantity that heme synthase can be produced because chloramphenicol causes a decrease in ALA synthase, heme synthatase, and DNA synthase. This can be associated with disease processes like sideroblastic anemia. What is the principle of the renish heavy metal test? What disease process is associated with lead poisoning? The principal of the renisch heavy metal is to place a clean coiled copper wire in a solution of 5-10 ml of gastric acid or urine with an equal amount of 2M HCL, then place in a hot water bath for 10 minutes, let sit one hour, and examine the copper wire for color change. ( blue or purple black- antimony; dull black- arsenic; shiny black- bismuth; and silver gray- mercury) Lead poisioning is associated with encephalopathy characterized by cerebral edema and hypoxia. How is carbon monoxide poisoning usually detected? This is usually detected by spot tests looking for carboxyhemoglobin (giving the patient a cherry red appearance due to its color), or gas chromatography also looking for carboxyhemoglobin. When does bromide toxicity result? How is it measured? How does bromide affect chloride determinations? This toxicity results from organic and inorganic medication. It is measured by immunoassay or thin layer chromotography, and it gives a false high in chloride determinations. Page 42

2.

3.

4.

5.

6.

What is the therapeutic usage of cyclosporine? methotrexate? Cyclosporin- this is an immunosuppressive drug that is used to suppress host vs. graft rejection of transplant organs. Methotrexate- this is an antineoplastic drug that is used in therapy and involves the rate of mitosis in normal cells versus neoplastic cells. What are digoxin? digitoxin? Why are their levels so critical? Digoxin- one of a group of cardiac glycosides obtained from digitalis plants which restores the force of cardiac contraction in congestive heart failure. (drug used to treat cardiovascular problems) Digitoxin- another cardiac glycoside (less common than digoxin) that is used to treat cardiovascular problems. Critical levelso Low- digoxin caused the atrium to be less electrically excitable o Moderate- reduce the rate of depolarization in the spontaneously depolarizing conductive fibers. o High- diminishes the depolarization of the ventricular myocardium. Define: therapeutic range- concentration range of a drug which is beneficial to the patient without being toxic. peak level- one hour after the dose is given when the drug reaches peak concentration in the body. trough level- the lowest concentration of drug obtained in the blood, drawn immediately prior to the next dose. toxic value- drug levels outside of the therapeutic range. bioavailability- tge fraction of a drug that is absorbed into the systemic circulation. What is theophyline? When is it used? A bronchodialator used to treat asthma or other chronic obstructive pulmonary diseases

7.

8.

9.

N. VITAMINS

1.

What disease process is associated with decreased B12 and folic acid? What is the relationship between B12 and folic acid? Pernicious anemia is associated with decreases B12 and folic acid. In relationship between B12 an folic acid, B12 is used in the metabolism and needed for the synthesis of folate which is needed for the production of nucleic acids (DNA)

Page 43

2.

What is the function of Vitamin A? How is it related to beta-carotene? How are both measured? Why are serum beta carotene levels measured? Vitamin A functions in growth, dim light vision, reproduction, immunity and mucous secretion. Beta-carotene(pro-vitamin A) is the precursor to vitamin A and is composed of two moles of vitamin A. They are both measured by immunoassay or HPLC. Beta carotene is measured in serum to indirectly quantitate 4. (P) Describe the methods for measuring Vitamin B12? What is the function of cyanocobalamin?

Page 44

BLOOD GASES

Purpose Represents the acid/base status of entire body Provides information of lung function Sample type Whole Blood Arterial Sample ABG Preferred sample Sites are radial, femoral or brachial artery Venous & Capillary Blood Can be used, but not preferred Assessment performed STAT

SPECIMEN COLLECTION & HANDLING


Collected

in heparinized plastic syringe (no air bubbles & no clots!!!) Collected by Respiratory Therapy

Often

Collected

anaerobically and put on ice. Ice serves to slow cell metabolism.

Testing

performed at 37o C, to emulate body temperature

PREANALYTICAL CONSIDERATIONS

Air bubbles
Causes increases in pO2, pH Causes decreased in pCO2

Clots

Can not run clotted whole blood on instrumentation

Glycolysis
Cell respiration causes a decrease in pH, pO2 pCO2 increases

Temperature

pH is temperature dependent. For every 1 degree rise in temperature, the pH decreases about 0.015 units

REFERENCE VALUES (ABG)


Component Arterial Blood Mixed Venous Blood

pH pO2 O2 Saturation pCO2 HCO3Total CO2 Base excess

7.35-7.45 80-100 mmHg > 95% 35-45 mmHg 22-26 mEq/L 23-27 mmol/L -2 to +2

7.31-7.41 35-40 mmHg 70-75% 41-51 mmHg 22-26 mEq/L 23-27 mmol/L -2 to +2

INSTRUMENTATION

Electrochemistry

Ion Selective Electrodes Spectrophotometry

Hemoglobin Concentration

DETERMINATION

Three components are directly measured


pH pO2 pCO2

Values that can be calculated and reported include:


Total CO2 or bicarbonate ion Base excess Oxygen saturation

PH

MEASUREMENT

Measure of the hydrogen ion activity based on bicarbonate-carbonic acid buffer system

pH electrode has a thin membrane of glass separating two differing H+ concentrations, a H+ exchange occurs in the outer layers of the glass, causing a potential to develop. A calomel half-cell or reference electrode is also immersed in the solution. Both the pH and reference electrode are connected through a pH meter. The meter can measure voltage difference between the two and convert to pH units.

PO2 MEASUREMENT Partial pressure of oxygen in the blood Measured by the O2 electrode to determine oxygen content pO2 electrode or Clark electrode measures the current that flows when a constant voltage is applied to the system As dissolved O2 diffuses from the blood a change in current occurs which offers a direct pO2 measurement

PCO2

MEASUREMENT

Partial pressure of carbon dioxide in the blood pCO2 measured in mmHg x 0.03 indicates carbonic acid (H2CO3) pCO2 > 50 mmHg = HYPO ventilation pCO2< 30 mmHg= HYPER ventilation

PCO2

MEASUREMENT

The pCO2 electrode or Severinghaus electrode consists of a pH electrode with a CO2 permeable membrane covering the glass surface. Between the two is a thin layer of dilute bicarbonate buffer. Once the blood contacts the membrane and the CO2 diffuses into the buffer, the pH of the buffer is lowered Change in pH is proportional to the concentration of dissolved CO2 in the blood

SiggaardAnderson nomogram

CALCULATED PARAMETERS

Siggaard-Anderson nomogram
Base Excess Total CO2 and bicarbonate concentration

BASE EXCESS
Determination of amount of base in the blood Determines the source of acid-base disturbance Base deficit usually indicates metabolic acidosis

Causes of:
Excess bicarbonate Deficit of bicarbonate

O2 SATURATION

Calculation/Derived

Requires measured pH and pO2 values Requires a hgb measurement usually obtained by cooximetry

Measured

Co-oximetry: measuring at multiple wavelengths to get light absorption spectra

REFERENCES

Bishop, M., Fody, E., & Schoeff, l. (2010). Clinical Chemistry: Techniques, principles, Correlations. Baltimore: Wolters Kluwer Lippincott Williams & Wilkins. Carreiro-Lewandowski, E. (2008). Blood Gas Analysis and Interpretation. Denver, Colorado: Colorado Association for Continuing Medical Laboratory Education, Inc. Jarreau, P. (2005). Clinical Laboratory Science Review (3rd ed.). New Orleans, LA: LSU Health Science Center. Sunheimer, R., & Graves, L. (2010). Clinical Laboratory Chemistry. Upper Saddle River: Pearson .

16

ELECTROLYTES

Electrolytes Substances whose molecules dissociate into ions when they are placed in water. Osmotically active particles Classification of ions: by charge

CATIONS (+)

In an electrical field, move toward the cathode Sodium (Na), Potassium (K), Calcium(Ca), Magnesium(Mg) In an electrical field, move toward the anode Chloride(Cl), Bicarbonate, PO4, Sulfate
2

ANIONS (-)

ELECTROLYTES
General

dietary requirements

Most need to be consumed only in small amounts as utilized Excessive intake leads to increased excretion via kidneys Excessive loss may result in need for corrective therapy

loss due to vomiting / diarrhea; therapy required - IV replacement, Pedilyte, etc.

ELECTROLYTE FUNCTIONS
Volume and osmotic regulation Myocardial rhythm and contractility Cofactors in enzyme activation Regulation of ATPase ion pumps Acid-base balance Blood coagulation Neuromuscular excitability Production of ATP from glucose

ELECTROLYTE PANEL

Panel consists of: sodium (Na) potassium (K) chloride (Cl) bicarbonate CO2 (in its ion form = HCO3- )

ANALYTES OF THE ELECTROLYTE PANEL


Sodium

(Na)

the major cation of extracellular fluid Most abundant (90 %) extracellular cation Diet

Easily absorbed from many foods

FUNCTION: SODIUM

Influence on regulation of body water Osmotic activity Sodium determines osmotic activity Main contributor to plasma osmolality Neuromuscular excitability extremes in concentration can result in neuromuscular symptoms Na-K ATP-ase Pump pumps Na out and K into cells Without this active transport pump, the cells would fill with Na+ and subsequent osmotic pressure would rupture the cells

REGULATION OF SODIUM

Concentration depends on: intake of water in response to thirst excretion of water due to blood volume or osmolality changes Renal regulation of sodium Kidneys can conserve or excrete Na+ depending on ECF and blood volume by aldosterone and the renin-angiotensin system

this system will stimulate the adrenal cortex to secrete aldosterone.


8

REFERENCE RANGES: SODIUM

Serum

136-145 mEq/L or mmol/L 40-220 mEq/L

Urine (24 hour collection)

SODIUM
Urine

testing & calculation:

Because levels are often increased, a dilution of the urine specimen is usually required. Once a number is obtained, it is multiplied by the dilution factor and reported as (mEq/L or mmol/L) in 24 hr.

10

DISORDERS OF SODIUM HOMEOSTASIS

Hyponatremia: < 136 mmol/L Causes of: Increased Na+ loss Increased water retention Water imbalance Hypernatremia:> 150 mmol/L Causes of: Excess water loss Increased intake/retention Decreased water intake

11

HYPONATREMIA
1.

Increased Na+ loss Aldosterone deficiency hypoadrenalism Diabetes mellitus In acidosis of diabetes, Na is excreted with ketones Potassium depletion K normally excreted , if none, then Na Loss of gastric contents
12

HYPONATREMIA
2.

Increased water retention Dilution of plasma Na+ Renal failure Nephrotic syndrome Hepatic cirrhosis Congestive heart failure
13

HYPONATREMIA
3.

Water imbalance Excess water intake Chronic condition

14

SODIUM
Note: Increased lipids or proteins may cause false decrease in results. This would be classified as artifactual/pseudo-hyponatremia

15

CLINICAL SYMPTOMS OF HYPONATREMIA

Depends on the serum level

Can affect
GI tract Neurological Nausea, vomiting, headache, seizures,coma

16

HYPERNATREMIA
1.

Excess water loss Sweating Diarrhea Burns Diabetes insipidus

17

HYPERNATREMIA
2.

Increased intake/retention
Excessive IV therapy Decreased water intake

3.

Elderly Infants Mental impairment

18

CLINICAL SYMPTOMS OF HYPERNATREMIA

Involve the CNS


Altered mental status Lethargy Irritability Vomiting Nausea

19

SPECIMEN COLLECTION: SODIUM


Serum (sl hemolysis is OK, but not gross) Heparinized plasma Timed and random urine Sweat GI fluids Liquid feces (would be only time of excessive loss)

20

ANALYTES OF THE ELECTROLYTE PANEL


Potassium

(K+)

the major cation of intracellular fluid


Only 2 % of potassium is in the plasma Potassium concentration inside cells is 20 X greater than it is outside. This is maintained by the Na-K pump exchanges 3 Na for 1 K

Diet

easily consumed by food products such as bananas

21

FUNCTION: POTASSIUM
Critically

important to the functions of neuromuscular cells Acid-base balance Intracellular fluid volume Controls heart muscle contraction Promotes muscular excitability Decreased potassium decreases excitability (paralysis and arrhythmias)
22

REGULATION OF POTASSIUM

Kidneys

Responsible for regulation. Potassium is readily excreted, but gets reabsorbed in the proximal tubule under the control of ALDOSTERONE

Diet Cell Uptake/Exchange

23

REFERENCE RANGES: POTASSIUM

Serum (adults)

3.5 - 5.1 mEq/L or mmol/L 3.7 - 5.9 mEq/L 25 - 125 mEq/L

Newborns

Urine (24 hour collection)

24

DISORDERS OF POTASSIUM HOMEOSTASIS

Hypokalemia < 3.5 mmol/L Causes of: Non-renal loss Renal Loss Cellular Shift Decreased intake Hyperkalemia >5.1 mmol/L Causes of Decreased renal excretion Cellular shift Increased intake Artifactual/False elevations

25

HYPOKALEMIA
1.

Non-renal loss Excessive fluid loss ( diarrhea, vomiting, diuretics ) Increased Aldosterone promote Na reabsorption K is excreted in its place

26

HYPOKALEMIA
2.

Renal Loss Nephritis, renal tubular acidosis, hyperaldosteronism, Cushings Syndrome Cellular Shift Alkalosis, insulin overdose Decreased intake

3.

4.

27

MECHANISM OF HYPOKALEMIA

Increased plasma pH ( decreased Hydrogen ion )

RBC H+ K+ K+ moves into RBCs to preserve electrical balance, causing plasma potassium to decrease. 28 ( Sodium also shows a slight decrease )

CLINICAL SYMPTOMS OF HYPOKALEMIA


Neuromuscular weakness Cardiac arrhythmia Constipation

29

HYPERKALEMIA
1.

Decreased renal excretion


Renal disease Addisons disease Hypoaldosteronism Such as acidosis, chemotherapy, leukemia, muscle/cellular injury Hydrogen ions compete with potassium to get into the cells

2.

Cellular Shift

30

HYPERKALEMIA
3.

Increased intake

Insulin IVs promote rapid cellular potassium uptake

4.

Artifactual Sample hemolysis Prolonged tourniquet use Excessive fist clenching

31

CLINICAL SYMPTOMS OF HYPERKALEMIA


Muscle weakness Tingling Numbness Mental confusion Cardiac arrhythmias Cardiac arrest

32

SPECIMEN COLLECTION:POTASSIUM
Non-hemolyzed serum heparinized plasma 24 hr urine

33

ANALYTES OF THE ELECTROLYTE PANEL


Chloride

(Cl-)

the major anion of extracellular fluid Chloride moves passively with Na+ or against HCO3- to maintain neutral electrical charge Chloride usually follows Na

if one is abnormal, so is the other

34

FUNCTION: CHLORIDE
Body

hydration/water balance Osmotic pressure Electrical neutrality

35

REGULATION OF CHLORIDE

Regulation via diet and kidneys


In the kidney, Cl is reabsorbed in the renal proximal tubules, along with sodium. Deficiencies of either one limits the reabsorption of the other.

36

REFERENCE RANGES: CHLORIDE

Serum 98 -107 mEq/L or mmol/L 24 hour urine 110-250 mEq/L varies with intake CSF 120 - 132 mEq/L Often CSF Cl is decreased when CSF protein is increased, as often occurs in bacterial meningitis.
37

DETERMINATION: CHLORIDE

Specimen type

Serum Plasma 24 hour urine CSF Sweat

Sweat Chloride Test Used to identify cystic fibrosis patients


Increased salt concentration in sweat Pilocarpine= chemical used to stimulate sweat production Iontophoresis= mild electrical current that stimulates sweat production

DISORDERS OF CHLORIDE HOMEOSTASIS

Hypochloremia Decreased blood chloride Causes of : Conditions where output exceeds input Hyperchloremia Increased blood chloride Causes of: Conditions where input exceeds output

39

HYPOCHLOREMIA

Decreased serum Cl
loss of gastric HCl salt loosing renal diseases metabolic alkalosis/compensated respiratory acidosis

increased HCO3- & decreased Cl-

40

HYPERCHLOREMIA

Increased serum Cl

dehydration (relative increase) excessive intake (IV) congestive heart failure renal tubular disease metabolic acidosis

decreased HCO3- & increased Cl-

41

SPECIMEN COLLECTION: CHLORIDE


Serum Heparinized plasma 24 hr urine Sweat

42

ANALYTES OF THE ELECTROLYTE PANEL

Carbon dioxide/bicarbonate (HCO3-) the major anion of intracellular fluid 2nd most abundant anion of extracellular fluid Total plasma CO2= HCO3- + H2CO3- + CO2

HCO3- (bicarbonate ion) accounts for 90% of total plasma CO2 H2CO3- (carbonic acid)
43

FUNCTION: BICARBONATE ION

CO2 is a waste product continuously produced as a result of cell metabolism, the ability of the bicarbonate ion to accept a hydrogen ion makes it an efficient and effective means of buffering body pH dominant buffering system of plasma

44

REGULATION OF BICARBONATE ION


Bicarbonate

is regulated by secretion / reabsorption of the renal tubules Acidosis : renal excretion Alkalosis : renal excretion

45

REGULATION OF BICARBONATE
ION

Kidney regulation requires the enzyme carbonic anhydrase which is present in renal tubular cells & RBCs
carbonic anhydrase

Reaction: CO2 + H2O H2CO3 H+ + HCO3

Pulmonary Control Renal Control


46

REFERENCE RANGE: BICARBONATE ION

Total Carbon dioxide (venous)

23-29 mEq/L or mmol/L

includes bicarb, dissolved & undissociated H2CO3 - carbonic acid (bicarbonate)

Bicarbonate ion (HCO3)

22-26 mEq/L or mmol/L

47

SPECIMEN COLLECTION: BICARBONATE ION


heparinized plasma arterial whole blood fresh serum Anaerobic collection preferred

48

ELECTROLYTE BALANCE

Anion gap an estimate of the unmeasured anion concentrations such as sulfate, phosphate, and various organic acids.

49

ELECTROLYTE SUMMARY

cations (+)
Na 142 K 5 Ca 5 Mg 2 154 mEq/L

anions (-)
Cl 105 HCO324 HPO422 SO4-2 1 organic acids 6 proteins 16

154 mEq/L

50

ANION GAP

Anion Gap Calculations


1.

Na - (Cl + CO2 or HCO3-)

Reference range: 7-16 mEq/L

Or

2.

(Na + K) - (Cl + CO2 or HCO3-)

Reference range: 10-20 mEq/L

51

FUNCTIONS OF THE ANION GAP

Causes in normal patients what causes the anion gap?

2/3 plasma proteins & 1/3 phosphate& sulfate ions, along with organic acids

Increased AG

uncontrolled diabetes (due to lactic & keto acids) severe renal disorders Hypernatremia lab error

Decreased AG 52

a decrease AG is rare, more often it occurs when one test/instrument error

REFERENCES

Bishop, M., Fody, E., & Schoeff, l. (2010). Clinical Chemistry: Techniques, principles, Correlations. Baltimore: Wolters Kluwer Lippincott Williams & Wilkins. http://thejunction.net/2009/04/11/the-how-to-authority-fordonating-blood-plasma/ http://www.nlm.nih.gov/medlineplus/ency/article/002350.htm Sunheimer, R., & Graves, L. (2010). Clinical Laboratory Chemistry. Upper Saddle River: Pearson .

53

Water Balance
Water 60% of total body mass Main Compartments Intracellular (ICF) inside cells 2/3 Extracellular (ECF) outside cells 1/3
2

Water Balance

Extracellular compartment

More on the ECF


Extracellular Compartment(ECF) Composed of two subcompartments Interstitial fluid (ISF) Surrounds cells in tissue Intravascular fluid (IVF) Volume of measurable fluid plasma

Body Fluid Composition


Plasma
55% of total blood volume Analytes measured directly Consists of ions, molecules, proteins

Serum

Water Balance
Ions exist in all of these fluids, but the concentration varies depending on individual ion and compartment The body uses active and passive(diffusion) transport principles to keep water and ion concentration in place

Water Balance
Plasma proteins ALBUMIN Draw water INTO the vessels Hydrostatic pressure Drives water OUT of the vessels These two forces create OSMOTIC or ONCOTIC PRESSURE

Water balance
Sodium has a pulling effect on water More Na outside cells than inside, the water is pulled out of cells into the extracellular fluid. Na+ determines osmotic pressure of extracellular fluid Proteins (especially albumin) inside the capillaries strongly pulls/keeps water inside the vascular system Albumin provides oncotic pressure. By keeping Na+ & albumin in their place, the body is able to regulate its hydration. When there is a disturbance in osmolality, the body responds by regulating water intake and urinary control of water loss or retention, not by changing electrolyte balance

Water Balance & Osmolality


Osmolality Physical property of a solution based on solute concentration
Water concentration is regulated by thirst and urine output Thirst and urine production are regulated by plasma osmolality

Water Balance & Osmolality


Increased osmolality stimulates two responses that regulate water

Hypothalamus stimulates the sensation of thirst Posterior pituitary secrets arginine vasopressin hormone (AVP)
AVP increases H2O re-absorption by renal collection ducts

In both cases, plasma water increases

10

Osmolality
Osmolality
concentration of solute / kg reported as mOsm / kg

another term:
Osmolarity - mOsm / L - not often used

11

Osmolality
Calculated osmolality
Formula:

uses glucose, BUN, & Na values


2 (Na) + glucose18 + BUN2.8 = calculated osmolality

Osmolal gap
Formula:

Difference between calculated and determined osmolality


Determined Osm/kg-calculated Osm/ kg= osmolal gap

Should be less than 10-15 units difference (


12

Formulas in Action
A 40-year-old woman suffers from vomiting and diarrhea. What would be her osmolality based on the below data? Sodium= 145 mmol/L Glucose= 750 mg/ dL BUN= 25 mg/dL

13

Regulation of Blood Volume


Renin-angiotension-aldosterone system
Aldosterone stimulates sodium reabsorption and potassium ion secretion

Natriuretic peptides Glomerular filtration rate Volume receptors


14

Renin-Angiotensin-Aldosterone System
Series of events Body detects decreased blood volume Renin converts angiotensinogen to angiotension I Angiotension I converted to angiotension II by ACE Angiotension II causes Vasoconstriction Secretion of aldosterone Stimulates AVP secretion and thirst Enhances NaCl reabsorption

15

References
Bishop, M., Fody, E., & Schoeff, l. (2010). Clinical Chemistry: Techniques, principles, Correlations. Baltimore: Wolters Kluwer Lippincott Williams & Wilkins. http://thejunction.net/2009/04/11/the-how-to-authorityfor-donating-blood-plasma/ http://www.nlm.nih.gov/medlineplus/ency/article/002350.h tm Sunheimer, R., & Graves, L. (2010). Clinical Laboratory Chemistry. Upper Saddle River: Pearson .

16

Overview of Iron
Essential mineral to most living organisms Most abundant trace element 2-2.5 of the 3-5 grams of iron in our bodies is found in hemoglobin (RBCs and RBC precursors)

Where does iron come from?


Dietary sources - meats, especially organ meats, spinach, beats,... etc.

Regulation
Dietary sources Absorption
Must be in ferrous state (Fe++) Occurs in the stomach/small intestines

Iron stores
Iron is recycled when RBCs are broken down 25% stored in liver, spleen and bone marrow as ferritin or (Fe3+)

Functions of Iron
Essential element of heme and hemoglobin Component of methemoglobin, myoglobin and some enzymes Cellular oxidative mechanisms

Heme Sythesis Review

The addition of ferrous iron (Fe++)forms heme

Forms of Iron
Ferrous(Fe2+) Ferric (Fe3+)
Absorbed form Ferritin Transport and storage form Free ferric form is picked up in the plasma by protein transferrin Delivered to cells having receptor sites

Gut mucosal cells Liver cells RE system cells Once inside the cell, ferric iron attaches to protein apoferritin to form ferritin Deficiency of apoferritin results in ferric iron deposits or hemosiderin, which is insoluble

Iron Links

http://www.umm.edu/blood/aneiron.htm http://www.ehendrick.org/healthy/000772.htm http://www.nlm.nih.gov/medlineplus/ency/article/000584.htm http://www.healthservices.gov.bc.ca/msp/protoguides/gps/ferritin.html

Hemoglobin
Structure, Synthesis, Degradation and Role
Refer to Hematology notes for review
Chapter 6 in McKenzie text

Porphyrins
General structure
Cyclic compounds called tetrapyrroles Linked by four pyrrole rings bonded by methene bridges

Porphyrins
Chemical intermediates in the synthesis of hemoglobin, myoglobin and other respiratory pigments (cytochromes) Clinical significance
Presence indicates abnormal heme synthesis

Physical properties
Color Coloration around 405 nm Usually red Fluorescence around 620 nm Reddish-pink color Chelation Arrangement of nitrogen atoms allows chelation of metal atoms such as iron, that participate in oxidative metabolism

Porphyrin Synthesis & Control


Synthesis Bone marrow and liver are the main site Some steps of synthesis occur in mitochrondria and cytoplasm of cell Control Enzyme: -aminolevulinic acid (ALA) Found in liver Increases in hepatic heme decrease the production of ALA Decreases or depletions of heme result in ALA increased production Rate of heme syntheis is flexible and can change rapidily in response to external stimuli

Porphyrins: Ones to keep an Eye on


Uroporphyrin: URO Water soluble Heme precursor Found in urine Coproporphyrin: COPRO Water soluble Heme precursor Found in urine and feces Protoporphyrin: PROTO Water insoluble Heme precursor Found in feces

Porphyrinogens
Reduced form of porphyrins Functional precursor of heme Difficult to measure due to instability and colorlessness

Glycated hemoglobin
Hemoglobin A 1c most stable Indicator of long-term glucose control
Why?
Reflects sustained average plasma glucose over the RBC life span

Correlates with risk of cardiovascular disease and other vascular disorders

Myoglobin
Heme protein found in skeletal and cardiac muscle Unable to release oxygen, except under low oxygen tension Main function is to transport oxygen from the muscle cell membrane to the mitochondria Serves as an extra reserve of oxygen to help exercising muscle maintain activity longer Used to diagnose acute myocardial infarction

Lead
Found in the environment and in paint Considered a toxin, plays no known role in NORMAL human physiology Exposure primarily respiratory or gastrointestinal Half-life in whole blood= 2-3 weeks Half-life= the time required by the body, tissue or organ to metabolize or inactivate half the amount of substance taken in

Lead
Absorption Depends on age, nutritional status and other substances that are present Transport Once in the blood, 94% transferred to RBC bound to hgb Once it reaches its half-life, lead is distributed to soft tissues, such as kidneys, liver and brain. Final storage is in soft tissue(5%) and bone (95%) Excretion Urine (76%) Feces (16%) Other (8%)

Specimen Requirements: Iron Studies


Serum without anticoagulant Plasma with heparin ( check product insert)

Oxalate, citrate or EDTA binds Fe ions, so they are unacceptable Early morning sample preferred due to diurnal variation

No hemolysis

Iron Study/Profiles

Three Components
Total Iron ( serum ) TIBC % Iron Saturation ( Fe Sat )

Total Iron % Fe Saturation = x 100 TIBC

The Iron Saturation is a measurement of how full transferrin is

Assessing Iron Levels and Forms

Directly measured
Iron Transferrin
Beta globulin formed in the liver Measured by the amount of iron it can bind

Ferritin
Best diagnostic test for IDA Acute phase reactant

Assessing Iron Levels and Forms

Indirect measure
TIBC (Total iron-binding capacity)
Measures the total amount of iron that apotransferrin can bind Can be expressed as a percentage(percent saturation)
Ratio of serum iron to TIBC

Increased
Late pregnancy IDA Following hemorrhage Following destruction of liver cells

Decreased
Decreased synthesis of transferrin Increased loss of urine proteins

Test Methodologies: Iron

Colorimetric Procedure
Separate Fe from transferrin with a strong acid Iron is reduced from ferrous(Fe3+) to ferric(Fe2+) state Addition of a chromogen creates a colored compound Measurement of colored product by spectrophotometry

Iron Reference Ranges

Diurnal variation Men: 65-165 g/dL Women: 45-160 g/dL

Decreased Levels
Decreased intake Increased need Increased loss

Increased Levels

Increased absorption Hemolytic anemia Lead poisoning Pernicious anemia Megaloblastic anemia Hepatitis

Test Methodologies:TIBC
Pre-treatment and Colorimetric Method 1. Add Fe3+ to saturate binding sites on transferrin 2. MgCO3 is added to remove unbound Fe3+ 3. Mixture is centrifuged and the supernatant tested using the serum iron methodology

Reference Ranges

Transferrin
200-360 mg/dL

Ferritin
Male: 20-250 ng/mL Female: 10-120 ng/mL

TIBC
250-425 g/dL

% saturation
15-55

Test Methodology: Hemoglobin

Electrophoresis
Discussed in separate unit

Test Methodology: Porphyrins

Screening tests
Urinary PBG Urinary ALA Urinayr porphyrins

Quantitative Assays
URO PROTO COPRO

Serve to classify porphyrias

Lab Methods

Watson-Schwartz for Urinary PBG( porphobilinogen)


Screen for acute intermittent porphyria Specimen
Qualitative: fresh morning urine Quantitative: 24 hour collection

Reference Range
<2 mg/daily

Watson-Schwartz

Principle
PBG + Ehrlichs reagent results in a red-orange chromogen Interferences
Urobilinogen indole

Lab Methods: HgbA1c


Electrophoresis Enzymatic Assays HPLC


Goal is to separate hemoglobin forms within a column. Then, glycated versus total hemoglobin can be measured spectrophotometrically Specimen requirements
EDTA whole blood Can be non-fasting

Reference range
4.0-6.0%

Lab Methods: Myoglobin


Procedures incorporate the binding of specific antibodies to myoglobin with a resulting chemical or physical change that can be measured and correlated to myoglobin concentration Specimen requirements

Usually plasma ( check product insert)

Specimen Requirements: Lead

Whole blood
Why? Circulating lead found in the RBC Venous sample preferred but capillary sample can be used ( must confirm positive on capillary) Royal blue top with EDTA anticoagulant Lead-free containers

Urine

Lab Method: Lead

Test methodologies
AAS Anodic stripping voltammetry

Reference Ranges in blood


Children< 10 g/dL

Iron Deficiency
Lab Features
Microcytic, hypochromic anemia Anisocytosis, poikilocytosis Total iron and Percent saturation decreased TIBC increased

Hemosiderosis
Excessive levels of iron in storage

Hemochromatosis
Characterized by an increased rate of absorption and less ferritin production Excessive iron deposits in organs Patient develops bronze color in the tissues Total iron, percent saturation increased TIBC decreased

Iron Status in Disease States


Condition
IDA Iron Overdose Hematochromatosis Malnutrition Chronic anemia Acute liver disease

Serum Iron
Decreased Increased Increased Decreased Decreased Increased

Transferrin
Increased Decreased Slight Decrease Decreased Normal/decrease d Variable

Ferritin
Decreased Increased Increased Decreased Normal/increase d Increased

% Saturation
Decreased Increased Increased Variable decreased Increased

Case Scenario #1
A 40-year-old female is scheduled to have an elective surgery. Her physician ordered a routine CBC pre-op. The following test results were obtained:
Test Hgb (g/dL) Hct (%) MCV (fL) MCHC (g/dL) WBC (x 103/L) Plts (x 109/L) Result 10 29.9 75 30 6.0 200 Reference Range 12-16.0 42-52 80-100 32-36 4.5-11 150-450

Case Scenario #1
On review of her blood smear, the technician noted target cells. What other types of morphology would we expect to see on this patient? The physician then ordered a serum iron, ferritin and TIBC level.

Case Scenario #1
Below are the results on the additional tests: What is her diagnosis?
Test Result Reference Range 65-165 20-200 260-440

Serum iron ( g/dL) 20 Ferritin ( g/dL) TIBC ( g/dL) 5 550

Hemoglobin Disorders
Refer to Hematology notes
Chapter 10: Hemoglobinopathies Chapter 11: Thalassemia

Porphyrin Disorders= Porphyrias


Inherited or Acquired Enzyme deficiencies resulting in overproduction of heme precursors in bone marrow or liver

Porphrias
Classification
Based on
Specific enzyme deficiency Hepatic vs erythropoietic Cutaneous vs neurologic

Porphyrias
Clinical symptoms
Cutaneous photosensitivity Itchy skin Hyperpigmentation Inflammatory reaction occurs on exposure to ultraviolet light Neurologic abnormalities due to increased ALA and PBG

Porphyrin Conditions
Secondary Conditions
Porphyrinuria Increase in coproporphyrin production Causes Lead intoxication Liver damage Infection Accelerated erythropoiesis Porphyrinemia Increase in erythrocytic protoporphyrin concentration Causes Lead intoxication Iron deficiency Impaired Iron absorption Chronic infection

Myoglobin
Elevations
Acute myocardial Renal failure Vigorous exercise Electric shock Intramuscular injections

LEAD
Clinical Features
Children
CNS symptoms: headache ,clumsiness, seizures, behavioral changes GI symptoms: Abdominal pain, colic, constipation

Adults
Peripheral neuropathies, motor weakness, anemia

Case Scenario #2
A mother brings her active 2-year-old son to the pediatrician for a routine visit. The physician orders a CBC. Below are the results:
Test Hgb (g/dL) Hct (%) Result 10.2 30.6 Reference Range 14-17.4 36-46

Case Scenario #2
The mother reports that her son has had some constipation and abdominal pain. The child does eat well, and the mother gives the child a vitamin supplement, which includes iron The mother did mention that they live in an older home that is in need of repainting. The physician orders further testing

Case Scenario #2
Results of testing
Test Serum iron Ferritin Whole blood lead (g/dL) Erythrocyte protoporphyrin (g/dL) Result 120 150 60 150 Reference Range 65-165 20-200 < 10 17-77

What is the diagnosis?


Lead Poisoning How does this occur? Lead inhibits certain enzymes in the heme synthesis pathway

Case Scenario #2
IDA was ruled out based on the serum iron and ferritin levels

Functions of the Heart


Pumps blood to the organs of the body Delivers oxygen and nutrients where they are needed Removes waste products from tissues

Symptoms of Heart Disease

Pathologic Conditions of the Heart


Congenital Cardiovascular Defects
Abnormality arises from abnormal formation of heart or its major blood vessels Present at birth
All defects develop before the 10th week of pregnancy

Origin unknown but appear to be based on genetic disposition and environmental influences

Congenital Cardiovascular Defects


Symptoms
Cyanosis Pulmonary hypertension Embolism Clubbed fingers Reduced growth Syncope

Examples
Tetralogy of Fallot Ventricular septal defects hole in the heart

Pathologic Conditions of the Heart


Heart Failure or Congestive Heart Failure Any structural or functional cardiac disorder that impairs the ability of the ventricle to fill with or eject blood Result Excess fluid accumulates in the lungs producing edema Reduced output of blood to systemic circulation Retention of fluid by the kidneys

Heart Failure or Congestive Heart Failure


Examples
Left ventricular dysfunction Coronary artery disease Cardiac arrhythmias

See it:
http://www.youtube.com/watch?v=3cW8__wFXDA

Pathologic Conditions of the Heart


Acute Coronary Syndromes
Term used to describe a series of events
Angina Reversible tissue injury Unstable angina Myocardial infarction Extensive tissue necrosis

Acute Coronary Syndromes


Clinical Symptoms
Chest pain Referred pain Nausea Vomiting Dyspnea Diaphoresis Light headedness

Acute Coronary Syndromes


Causes
Atherosclerosis
Inflammatory disorder Plaques deposit in artery walls Leads to ischemia

Stages of Atherosclerosis
1. Initial vascular injury caused by: 1. Hypertension, hyperlipidemia, hyperhomocysteinemia 2. Increased permeability to lipids especially LDL/VLDL 1. Results in inflammation 3. Monocytes & Leukocytes arrive to help! 4. Macrophages scavenge LDL/cholesterol-rich lipoproteins- become foam cells 5. Foam cells promote lesion progression 6. T and B lymphocytes are recruited by the plaque 7. Interactions between T/B lymphs and foam cells recruits smooth muscle cells into the lumen 8. Smooth muscle cells secrete collagen, elastin, and proteoglycans to fix the plaque to the vessel wall See the process

Presentation of Coronary Heart Disease

Hypertension
Persistent systolic BP of at least 140 mm HG and/or diastolic BP of at least 90 mm Hg Prevalence increases with age Contributing factors
Obesity Physical inactivity Unhealthy nutrition

Hypertensive Heart Disease


Term used to describe heart disease caused by direct or indirect effects of increased BP Peripheral resistance determining factor in BP
Increases workload of left ventricle resulting in hypertrophy and dilation of mitral valve. This valve is affected and blood is regurgitated to the left atrium

Infective Heart Disease


Heart disease caused by infectious agents Examples
Rheumatic Heart Disease
Complication of rheumatic fever due to autoimmune response. Causative organism is Group A streptococcus Usually affects young adults and children

Infective Endocarditits
Infection of endocardial surface of the heart Causative organism Group D streptococcus, but others also

Pericarditis
Inflammation of the pericardium Causative agents include bacteria, fungi, viral, autoimmune, others

Diagnosing Heart Disease


Myocardial Infarction
Diagnosis based on clinical symptoms, EKG changes and the rise/fall of biochemical markers Samples collected at onset, 6-9 hours and 12-24 hours if previous samples were negative Preferred biomarkers are Troponin I and T.
Specific and sensitive for myocardial necrosis Current guidelines suggest the use of 2 markers for diagnosis

Current Cardiac Panel


Myoglobin
Released from damaged cardiac/skeletal muscle

Cardiac troponins
See upcoming slide

CK CK-MB BNP
Discussed later

Time Course Of Enzyme Activity in MIs


Historically CK, CK-MB, AST, LD/LDH isoenzymes used
Enzyme Onset of Elevation Peak activity (Hr) Duration of Elevation

CK CK-MB AST LD
LDH isoenzymes

4-8 4-6 8-12 12-24 12-24

12-24 12-24 24 72

3-4 2-3 5 10 5

Time Course Of Enzyme Activity in MIs


Troponin
Rises 4-10 hours after onset Peak at 12-48 hours Elevated for 4-10 days Released 1-4 hours after onset Rises within 4-6 hours after onset Peaks at 12-24 hours Normal at 2-3 days

Myoglobin CK-MB

Troponin
Consists of three proteins that bind to thin filament(actin) of cardiac and skeletal muscle Troponin T (TnT) Troponin I (TnI) Troponin C (TnC) Function to bind Ca+ and regulate muscle contraction Absent in the serum of healthy people

Whats So Special About Troponin?


Specific for cardiac tissue High diagnostic specificity and sensitivity Early detection following MI Remain elevated for several days Undetected in healthy people Few interfering substances in detection

Markers of Inflammation
High Sensitivity C-Reactive Protein (hsCRP)
Acute phase protein Produced in the liver in response to injury, infection and inflammation Increases in CRP correlate with the risk of coronary artery disease

Markers of Congestive Heart Failure


Natriuretic peptide
Hormones that include atrial natriuretic peptide (ANP), B-type natriuretic peptide(BNP), C-type and Dtype Assist in regulation of cardiovascular homeostasis BNP
Released on ventricular stretch or stress to myocytes in the absence of necrosis Increased BNP indicates expanded fluid volume such as that seen in renal failure and CHF

Vascular Inflammation

Plaque Destabilization

Plaque Rupture

Acute Phase Reactant (CRP)

Ischemia

Necrosis (Troponin)

Myocardial Dysfunction (BNP, NT-proBNP)

References
Bishop, M., Fody, E., & Schoeff, l. (2010). Clinical Chemistry: Techniques, principles, Correlations. Baltimore: Wolters Kluwer Lippincott Williams & Wilkins. http://medicinembbs.blogspot.com/2010/12/congestiveheart-failure.html http://www.resverlogix.com/product_development/cardiovas cular_disease/atherosclerosis.html http://www.youtube.com/watch?v=upb37rbS1dE http://www.youtube.com/watch?v=3cW8__wFXDA http://smabiology.blogspot.com/2009_04_01_archive.html Sunheimer, R., & Graves, L. (2010). Clinical Laboratory Chemistry. Upper Saddle River: Pearson .

Introduction
2

Organisms rely on the oxidation of complex

organic compounds to obtain energy chemical energy to our cells


Lipids=Fats Amino acids = Proteins Carbohydrates= Sugars, starches

Three general types of compounds provide

Carbohydrates
3

Major food source & energy supply of body Primary source of energy for brain, erythrocytes, retinal cells

Depending on individual diet, 50-90% of the body's carbohydrate intake is in the form of

Grains - cereals, bread Starchy vegetables - potatoes Legumes - beans, peanuts other sources = sugar, molasses, lactose from milk, fructose from fruit

Stored primarily as liver and muscle glycogen

Description and Classification of Carbohydrates


Contain C, H and O molecules Contain a C=O (ketone) and an OH(aldehyde)

functional group

Classification Based on certain properties


The size of the base carbon chain Location of the CO functional group Number of sugar units Stereochemistry of compound

Chemical Properties
5

Some ( not all ) carbs are reducing substances (donate electrons)


Chemical reduction of other substances These sugars must contain an aldehyde or ketone group Reducing sugars Glucose Maltose Lactose Fructose Galactose

Sucrose is not a reducing substance

Carbohydrate Metabolism
Glucose is primary energy source Nervous tissue can not concentrate or store

carbohydrates, so a steady supply of glucose is needed Once the level of glucose falls below a certain range, normal function is impaired

Carbohydrate Breakdown
Mouth Salivary amylase

Dietary Carbohydrates

Dextrins/ Maltose

Stomach/Intestines Pancreatic amylase

Absorption into intestinal mucosa Delivered to liver


Monosaccharide

Carbohydrate Breakdown
Ultimate Goal Convert glucose to CO2 and water with ATP as a by-product Possible channels Converted to liver glycogen and stored Metabolized to CO2 and H2O Converted to keto-acids, amino acids, and proteins Converted to fats and stored in adipose tissue

Biochemical Pathways in Carbohydrate Breakdown

Embden-Meyerhoff pathway
Converts glucose to pyruvate/lactate Primary energy source for humans

Hexose monophosphate shunt


Oxidizes glucose to ribose and CO2 Produces NADPH as an energy source

Glycogenesis

Converts glucose to glycogen

Carbohydrate Metabolism
10

Glycolysis the conversion of glucose and other hexoses into

lactate or pyruvate Breakdown of glucose for energy production

Glycogenesis the conversion of glucose to glycogen

usually in liver & muscle Excess glucose is converted and stored as glycogen High concentrations of glycogen in liver and skeletal muscle Glycogen is a quickly accessible storage form of glucose

Carbohydrate Metabolism
11

Glycogenolysis the breakdown of glycogen to form glucose

Glycogenolysis occurs when plasma glucose is decreased Occurs quickly if additional glucose is needed Controlled by hormones & enzymes

Gluconeogenesis the formation of glucose from non-carbohydrate

sources, such as amino acids, glycerol & fatty acids into glucose Occurs mainly in the liver

Glycolysis

Gluconeogenesis

Glucose

Glycogenolysis

Glycogenesis

Carbohydrate Metabolism
13

Also related: Lipogenesis the conversion of carbohydrates to fatty acids

Fat is another energy storage form, but not as quickly accessible as glycogen

Lipolysis the decomposition of fat

The sum or net of all of these processes determines

the level of blood glucose.

Regulation of Plasma Glucose


14

Organs / systems involved in glucose regulation

Liver : Muscle

Glucose

Glycogen

Glucose

Skeletal & heart

Pancreas Synthesizes hormones Insulin and Glucagon, somatostatin

Other Endocrine glands


Anterior pituitary gland ( growth hormone) Adrenal gland (epinephrine and cortisol) Thyroid gland (thyroxine)

Regulation of Plasma Glucose


15

If plasma glucose is decreased :


Glycogenolysis

The liver releases glucose into the plasma (quick response)

Gluconeogenesis and lipolysis

If plasma glucose is increased :


Glycogenesis

Liver stores glucose as glycogen Formation of lipids

Lipogenesis

Hormones that Regulate Glucose


16

Insulin Most important & only one to decrease glucose level Synthesized in the Beta cells of the Islets of Langerhans (in the pancreas) Released when plasma glucose is increased

Action / Effects of insulin


Facilitates glucose entry into cells

cell membranes need insulin to be present for glucose to enter glucose to glycogen

Promotes liver glycogenesis

Promotes glycolysis

speeds up utilization of glucose in cells Such as the formation of Triglycerides

Promotes synthesis of lipids from glucose

Promotes amino acid synthesis from glucose intermediates Decreases / inhibits glycogenolysis and gluconeogenesis

Insulin Control
18

Insulin secretion controlled by:


Blood glucose level Certain Amino Acids ie. leucine, & arginine

Counterregulatory Hormones
19

Glucagon 2nd most important glucose regulatory hormone

Referred to as a hyperglycemic agent Synthesized in alpha cells of the islets of Langerhans

Action/Effect of Glucagon
20

Stimuli decreased plasma glucose Action Increases glycogenolysis & gluconeogenesis Promotes breakdown of fatty acids Promotes breakdown of proteins to form amino acids Increases plasma glucose concentration

Other Regulatory Hormones


21

Epinephrine

One of two glucose regulating hormones from the adrenal gland Origin adrenal medulla Action/effect Inhibits insulin secretion & release Promotes lipolysis Stimulates glycogenolysis Immediate release of glucose Stimuli Neurogenic - based on physical / emotional stress. Adrenal tumors

Other Regulatory Hormones


22

Glucocorticoids - such as cortisol Origin adrenal cortex

Effect antagonistic to insulin


increases blood glucose promotes gluconeogenesis from breakdown of proteins inhibits the entry of glucose into muscle cells

Stimuli anterior pituitarys ACTH

Other Regulatory Hormones


23

Growth Hormone (GH) and Adrenocorticotropic

Hormone (ACTH)

Origin anterior pituitary gland Effect antagonistic to insulin

Increases plasma glucose levels inhibits insulin secretion inhibits entry of glucose into muscle cells inhibits glycolysis inhibits formation of triglycerides from glucose

Stimuli
decreased glucose stimulates its release increased glucose inhibits its release

Other Regulatory Hormones


24

Thyroid hormones (such as thyroxine) Origin thyroid gland Effect


increases absorption of glucose from intestines Promotes comversion of liver glycogen to glucose

Stimuli pituitary glands TSH

Other Regulatory Hormones


Somatostatin Origin-Delta cells of the islets of Langerhans in the pancreas Effect - increase plasma glucose Actions
antagonistic to insulin, inhibits endocrine hormones including glucagon & growth hormone

References
Bishop, M., Fody, E., & Schoeff, l. (2010). Clinical Chemistry:

Techniques, principles, Correlations. Baltimore: Wolters Kluwer Lippincott Williams & Wilkins. Sunheimer, R., & Graves, L. (2010). Clinical Laboratory Chemistry. Upper Saddle River: Pearson .

26

Laboratory testing

Considerations
Reference values depend on: Type of specimen
venous/capillary Serum, plasma, whole blood

How was it collected?


fasting, random, after a meal

Reference value (serum/plasma)


74-106 mg/dL

Laboratory testing

Glucose preservation
Perform testing < 1 hour after collection Separate plasma from cells < 1 hour
Cells continue to utilize glucose at a rate of 10 mg/dL per hour. Refrigeration slows the process.

Collect blood in sodium fluoride tube


Grey top tube Fluoride inhibits glycolysis

Specimen Collection

Whole blood
Point of care Results are @ 11% lower than plasma/serum

Serum Plasma

Other Specimen Types

CSF specimens
Analyzed ASAP Glucose level is 60-70% of pts current blood level. CSF glucose in Fasting (non-diabetic) @ 40-70 mg/dL
Decreased CSF glucose values suggest bacterial meningitis because bacteria are consuming glucose as an energy source Normal or Increased CSF glucose suggests viral meningitis.

24 hour urine
A small amount of glucose is lost in the urine daily. Usually < 500mg/24 hr. Random urine for diagnosis no longer performed, but some patients use it for self monitoring.
5

Methods for Glucose Determination

Glucose Oxidase Methodology


Trindler reaction Glucose Oxidase Peroxidase Gluconic acid + H2O2 Oxidized chromogen + H2O

Glucose + O2 + H2O H2O2 + Chromogen

Glucose oxidase an enzyme that will catalyze the reaction of glucose to gluconic acid, with the formation of hydrogen peroxide as a by-product

Glucose oxidase

Good methodology, but:


Procedure is good for blood and CSF specimens, but urine has too many interfering substances. Subject to interference from ascorbic acid, bilirubin and uric acid which are also oxidized by peroxidase.

Alternative way to determine concentration: (polarographically)


Measuring the amount of oxygen used up by an electrode

Hexokinase

An enzyme that catalyzes the phosphorylation of glucose


Method can be very accurate and precise since the coupling reaction is specific Time consuming for routine use Reference methodology since it lacks interferences associated with glucose oxidase method Procedure can utilize blood, urine and CSF

Hexokinase Methodology

Glucose + ATP Glucose 6 - Phosphate + NADP

Hexokinase G6PD

Glucose 6 Phosphate + ADP NADPH + H + 6-Phosphogluconate

NADP - Nicotinamide adenine dinucleotide phosphate (oxidized form) is reduced NADPH - reduced form absorbs light (340nm) proportional to the amount of glucose present in first reaction

10

Laboratory Diagnosis

Laboratory Tests

Fasting blood sugar (FBS)


Most frequently ordered screening test for glucose metabolism
Reference value: 74-106 mg/dL Fasting values > 126 mg/dL usually indicate a problem FBS should be repeated on another day to confirm diagnosis Borderline diabetes may have a normal FBS & may need a challenge test to demonstrate abnormality
12

Laboratory Tests

2 Hour Postprandial
Patient has FBS drawn Ingests a 75 gram high carbo breakfast or sometimes drinks glucola Has repeated glucose test at 2 hours Glucose level should have returned to fasting levels. If glucose > 200 mg/dL on the postprandial test, a fasting or random glucose level, should be performed on a subsequent day to diagnose with diabetes
13

Laboratory Tests

Oral glucose tolerance test (GTT)


No longer recommended by the new ADA guidelines Used to screen for gestational diabetes Problems included calculation dosage, patient must drink it, keep it down, stay relatively inactive during test period, and be successfully drawn on time.

14

Oral glucose tolerance test (GTT)

Patient directions - important.


Eat an adequate carbohydrate diet at least three (3) days prior to test Evening before the test, no eating after supper meal Test is begun in early a.m. Obtain fasting specimen Test dose: ** test dose has been reduced to 75 gm for adults and 1.75 gm / kg for children. Test dose must be consumed within 5 minutes. Patient is to remain resting, no smoking or eating during test period Blood and urine specimens are collected at hourly intervals - Testing of the urine glucose & ketones, no longer routine.

15

Oral glucose tolerance test (GTT)

Response to Oral Glucose Tolerance Test

Abnormal

Normal

Laboratory Tests: Ketones


Produced by the liver Metabolism by-products of fatty acids Three bodies
Acetone (2%) Acetoacetic acid (20%) 3- hydroxybutyric acid (78%)

Increase in cases of carbohydrate deprivation or decreased carbohydrate use (diabetes mellitus, starvation/fasting, prolonged vomiting etc.)

Laboratory Tests: Microalbumin

Microalbumin
Persistent albuminuria in the range of 30-299 mg/ 24 h or an albumin-creatinine ratio of 30300 g/mg Indication of renal nephropathy Assists in the diagnosis of early proteinuria Normal urine dipsticks are insensitive to low concentrations of urine albumin

Glycosylated Hemoglobin/ Hemoglobin A1c

Long term glycemic control indicator, reflects average blood glucose level over the previous 2-3 months Glucose molecule attaches nonenzymatically to the hemoglobin molecule

Advantages:

Time average glucose not subject to temporary variability due to diet and exercise Does not require fasting Conditions that affect the life span of the RBC, such as sickle cell disease and hemolytic diseases Hemoglobin A1C is the most commonly measured glycosylated hemoglobin
19

Influenced by:

Glycosylated Hemoglobin/ Hemoglobin A1c

Specimen : EDTA whole blood doesnt need to be fasting Measured by electrophoresis, enzymatic assays, HPLC Hemoglobin A1C reference range 4.0 - 6.0 % For diagnosis of diabetes based on Hemoglobin A1C results, the patient must has a result of > 6.5% , confirmed by repeat measurement.

Other related tests: Lactose Tolerance


Lactose - disaccharide Lactose malabsorption or lack of enzyme needed to breakdown lactose Often results in diarrhea, cramping, and gas Lab evaluation
Perform OGTT using lactose, not glucose

Normal

Lactase deficiency

GTT curve similar to OGTT (glucose level will increase 25 mg/dL above the fasting level). Flat curve - no/very little increase in glucose level.
21

Urine Glucose

Copper Reduction- Clinitest

Not specific Detects all reducing sugars Used to detect galactosemia in babies and children < 3 yrs old.

References

Bishop, M., Fody, E., & Schoeff, l. (2010). Clinical Chemistry: Techniques, principles, Correlations. Baltimore: Wolters Kluwer Lippincott Williams & Wilkins. Sunheimer, R., & Graves, L. (2010). Clinical Laboratory Chemistry. Upper Saddle River: Pearson .

23

Hyperglycemia
Increase in plasma glucose levels due to hormone imbalance Healthy patients Insulin is secreted by the cells of the pancreatic islets of Langerhans Reference Range Increased plasma glucose: Glucose reference range:
74 - 106 mg / dl > 110 mg / dl

Effects of Hyperglycemia
Immediate Effects Increased extracellular osmotic pressure The increased glucose in plasma pulls water out of cells Results in dehydration Acidosis - metabolic acidosis. May result If the patients cells are not able to take in glucose, they may begin to convert fats to fatty acids, which then become keto acids.

Effects of Hyperglycemia: Long term


Physiological Heart attacks/strokes, Diabetic retinopathy(Blindness), kidney failure, neurologic defects, susceptibility to infections Chemical Glycosylated hemoglobin
the formation of glycosylated hemoglobin is the result of prolonged elevation of plasma glucose.

Diabetes
Characterized by hyperglycemia Disorders differ in etiology, symptoms and consequences Labs role
Assist in diagnosis of the disease Identification of the disorder Assessment of progression of tissue damage

Physiologic abnormalities of diabetes


Hyperglycemia increase blood glucose. Doesnt matter how the glucose is derived - diet, fat metabolism, protein destruction/wasting Ketosis from fat metabolism, ketonemia, ketonuria Hyperlipidemia -increase blood lipids from faulty glucose metabolism. Decrease blood pH - metabolic acidosis Urine abnormalities Glycosuria glucose present Polyuria - increase in urine volume Loss of electrolytes - washing out with the urine
6

Diabetes
World Health Organization (WHO) and American Diabetes Association (ADA) recommends four categories of diabetes: Type 1 diabetes Most severe and potentially lethal Type 2 diabetes Other (secondary diabetes) Gestational diabetes mellitus (GDM)

Type 1 Diabetes
Insulin dependent diabetes mellitus ( IDDM ) 5-10 % of diabetes cases Demographics
Non-Hispanic Whites/ Non-Hispanic Blacks Children & adolescents

Pathology

Disease triggered by viral illness or environmental factors that destroys beta cells in pancreas. Absolute Insulin deficiency Defect in secretion, production or action or all Autoimmune destruction of islet beta cells in pancreas Auto-antibodies are present

Type 1 Diabetes
Clinical Symptoms
CLASSIC TRIAD Polyphagia (increased food uptake) Polydipsia (thirst) Polyuria ( increased urine production) Other symptoms Mental confusion Rapid weight loss Hyperventilation Diabetic ketoacidosis
9

Laboratory Findings
Hyperglycemia- plasma levels > 110 mg/dL Glucosuria- plasma glucose > 180 mg / dl Decreased insulin Increased glucagon Stimulation causes Gluconeogenesis Lipolysis (breakdown of fat produces ketones) Ketoacidosis Decreased blood pH ( acidosis ) Sodium Potassium CO2

10

Type II Diabetes
Non Insulin Dependent Diabetes Mellitus( NIDDM ) Most common form of diabetes Demographics Adult onset Patients usually > 20 years old American Indians and non-Hispanic blacks

11

Type II Diabetes: Pathology


Develops gradually Disorder in insulin resistance and relative deficiency of insulin Plasma glucose is unable to enter cells Contributory factors Obesity Lack of exercise Diet Genetics Drugs, such as diuretics, psychoactive drugs Increases in hormones that inhibit/antagonize insulin (GH & cortisol)
12

Laboratory Findings
Hyperglycemia Glucosuria Insulin is present Glucagon is not elevated No lipolysis and no ketoacidosis Excess glucose is converted to triglycerides ( plasma triglycerides ) Normal / Increased Na / K Increased BUN & Creatinine ( Decreased renal function ) Hyperosmolar plasma from hyperglycemia

13

Other (SecondaryDiabetes)
Genetic defects of beta cell function Genetic defects in insulin action Genetic syndromes Pancreatic disease Endocrinopathies Drug or chemical induced

14

Gestational Diabetes
Glucose intolerance associated with pregnancys hormonal and metabolic changes Mothers usually return to normal after pregnancy, but with increased risk for diabetes later on in life Infants are at increased risk for respiratory complications and hypoglycemia after birth

15

Criteria for Diagnosis of Diabetes


1. Symptoms of diabetes plus random plasma glucose concentration > 200 mg/dL. Random is defined as any time of day without regard to time OR 2. Fasting plasma glucose > 126 mg/dL. Fasting is defined as no caloric intake for at least 8 hours. OR 3. 2-Hour postprandial glucose > 200 mg/dL during an oral glucose tolerance test OR 4. A HgbA1C > 6.5%, confirmed on repeat measurement Side notes Glucose tolerance testing ( GTT ) is considered to be of limited additional use in the diagnosis of diabetes and not recommended, do 2 hour pp test as stated above. Urine glucose testing is also not recommended in diabetes diagnosis

16

Hypoglycemia
Plasma glucose level falls below 60 mg/dL Glucagon is released when plasma glucose is < 70 mg / dL to inhibit insulin Epinephrine, cortisol, and growth hormone released from adrenal gland to increase glucose metabolism and inhibit insulin Treatment Varies with cause. Generally, hypoglycemia is treated with small, frequent meals, (5-6 / day) low in carbohydrates, high in protein
17

Hypoglycemia
Symptoms Increased hunger Sweating Nausea Vomiting Dizziness Shaking Blurring of speech and sight Mental confusion Lab Findings Decreased plasma glucose
Whipples Triad Symptoms of hypoglycemia Low plasma glucose at time of symptoms Alleviation of symptoms with glucose ingestion

18

19

Hypoglycemia
Causes of: Reactive Fasting
Insulin overdose in diabetics Ethanol ingestion Insulin-producing tumors Hepatic dysfunction Sepsis

20

Galactosemia
Resulting from :
Galactose 1, phosphate uridyl transferase deficiency enzyme that converts galactose to glucose, patients cannot change either galactose or lactose into glucose. results in galactosemia (galactose in blood)

Effects:
Can lead to mental retardation, cataracts, death

check children < 3 yrs for reducing substances

21

References
Bishop, M., Fody, E., & Schoeff, l. (2010). Clinical Chemistry: Techniques, principles, Correlations. Baltimore: Wolters Kluwer Lippincott Williams & Wilkins. Centers for Disease Control. (2012). Diabetes Public Health Resource. Retrieved from http://www.cdc.gov/diabetes/pubs/factsheet11.htm Sunheimer, R., & Graves, L. (2010). Clinical Laboratory Chemistry. Upper Saddle River: Pearson . http://crossfitovercome.com/2011/12/29/diabetes-primer/

22

Terms

Acid Any substance that can yield a hydrogen ion (H+) or hydronium ion when dissolved in water Release of proton or H+ Base Substance that can yield hydroxyl ions (OH-) Accept protons or H+

Terms

pK/ pKa Negative log of the ionization constant of an acid Strong acids would have a pK <3 Strong base would have a pK >9 pH

Negative log of the hydrogen ion concentration pH= pK + log([base]/[acid]) Represents the hydrogen concentration

Terms

Buffer

Combination of a weak acid and /or a weak base and its salt What does it do?

Resists changes in pH

Effectiveness depends on

pK of buffering system pH of environment in which it is placed

Terms

Acidosis

pH less than 7.35

Alkalosis

pH greater than 7.45

Note: Normal pH is 7.35-7.45

Acid-Base Balance

Function Maintains pH homeostasis Maintenance of H+ concentration Potential Problems of Acid-Base balance Increased H+ concentration yields decreased pH Decreased H+ concentration yields increased pH

Regulation of pH

Direct relation of the production and retention of acids and bases Systems Respiratory Center and Lungs Kidneys Buffers Found in all body fluids Weak acids good buffers since they can tilt a reaction in the other direction Strong acids are poor buffers because they make the system more acid

Blood Buffer Systems

Why do we need them?

If the acids produced in the body from the catabolism of food and other cellular processes are not removed or buffered, the bodys pH would drop Significant drops in pH interferes with cell enzyme systems.

Blood Buffer Systems

Four Major Buffer Systems

Protein Buffer systems


Amino acids Hemoglobin Buffer system

Phosphate Buffer system Bicarbonate-carbonic acid Buffer system

Blood Buffer Systems

Protein Buffer System

Originates from amino acids

ALBUMIN- primary protein due to high concentration in plasma

Buffer both hydrogen ions and carbon dioxide

Blood Buffering Systems

Hemoglobin Buffer System Roles Binds CO2 Binds and transports hydrogen and oxygen Participates in the chloride shift Maintains blood pH as hemoglobin changes from oxyhemoglobin to deoxyhemoglobin

Oxygen Dissociation Curve


Curve B: Normal curve Curve A: Increased affinity for hgb, so oxygen keep close Curve C: Decreased affinity for hgb, so oxygen released to tissues

Bohr Effect

It all about oxygen affinity!

Blood Buffer Systems

Phosphate Buffer System Has a major role in the elimination of H+ via the kidney Assists in the exchange of sodium for hydrogen It participates in the following reaction HPO-24 + H+ H2PO 4

Essential within the erythrocytes

Blood Buffer Systems

Bicarbonate/carbonic acid buffer system


Function almost instantaneously Cells that are utilizing O2, produce CO2, which builds up. Thus, more CO2 is found in the tissue cells than in nearby blood cells. This results in a pressure (pCO2). Diffusion occurs, the CO2 leaves the tissue through the interstitial fluid into the capillary blood

Bicarbonate/Carbonic Acid Buffer


Excreted by lungs

Carbonic acid Conjugate base

Bicarbonate

Excreted in urine

Bicarbonate/carbonic acid buffer system

How is CO2 transported? 5-8% transported in dissolved form A small amount of the CO2 combines directly with the hemoglobin to form carbaminohemoglobin 92-95% of CO2 will enter the RBC, and under the following reaction

CO2 + H20

H+ + HCO3-

Once bicarbonate formed, exchanged for chloride

Henderson-Hasselbalch Equation

Relationship between pH and the bicarbonate-carbonic acid buffer system in plasma Allows us to calculate pH

Henderson-Hasselbalch Equation

General Equation

pH = pK + log A-

HA

Bicarbonate/Carbonic Acid system


o

pH= pK + log HCO3 H2CO3 ( PCO2 x 0.0301)

Henderson-Hasselbalch Equation
1.

2.

pH= pK+ log H HA The pCO2 and the HCO3 are read or derived from the blood gas analyzer pCO2= 40 mmHg HCO3-= 24 mEq/L Convert the pCO2 to make the units the same pCO2= 40 mmHg * 0.03= 1.2 mEq/L Lets determine the pH: Plug in pK of 6.1 Put the data in the formula pH = pK + log 24 mEq/L 1.2 mEq/L pH = pK + log 20 pH= pK+ 1.30 pH= 6.1+1.30 pH= 7.40

3.

3. 4.

5.

The Ratio.
Normal is : 20 = Bicarbonate = Kidney = metabolic 1 carbonic acid Lungs respiratory

The ratio of HCO3- (salt/bicarbonate) to H2CO3 (acid/carbonic acid) is normally 20:1 Allows blood pH of 7.40 The pH falls (acidosis) as bicarbonate decreases in relation to carbonic acid The pH rises (alkalosis) as bicarbonate increases in relation to carbonic acid

Physiologic Buffer Systems

Lungs/respiratory

Quickest way to respond, takes minutes to hours to correct pH by adjusting carbonic acid Eliminate volatile respiratory acids such as CO2 Doesnt affect fixed acids like lactic acid Body pH can be adjusted by changing rate and depth of breathing blowing off Provide O2 to cells and remove CO2

Physiologic Buffer Systems

Kidney/Metabolic

Can eliminate large amounts of acid Can excrete base as well Can take several hours to days to correct pH Most effective regulator of pH

If kidney fails, pH balance fails

25

References

Bishop, M., Fody, E., & Schoeff, l. (2010). Clinical Chemistry: Techniques, principles, Correlations. Baltimore: Wolters Kluwer Lippincott Williams & Wilkins. Carreiro-Lewandowski, E. (2008). Blood Gas Analysis and Interpretation. Denver, Colorado: Colorado Association for Continuing Medical Laboratory Education, Inc. Sunheimer, R., & Graves, L. (2010). Clinical Laboratory Chemistry. Upper Saddle River: Pearson .

26

You might also like